Contracts Quiz Questions

Pataasin ang iyong marka sa homework at exams ngayon gamit ang Quizwiz!

A music promoter planned to open a discotheque and bar on the outskirts of town. He hired a builder to build it, with a completion date of September 15. Although the promoter was very optimistic that the disco would be a big success, its profitability could not be determined with certainty. First year profits were estimated to be about $1,000 per day. To encourage the builder to work in a timely manner, the contract included a liquidated damages clause, providing that the builder would pay the promoter $10,000 per day for each day the contract ran over its completion date. The builder's work progressed smoothly and would have been finished on time, except that the builder failed to place in a timely manner his order for the disco's specially manufactured dance floor lighting. Consequently, the work was not completed until October 15. The promoter sued the builder for breach of contract. The builder called a witness who testified that the disco would have received $30,000 in income during that 30-day period and would have expended $20,000, leaving a profit of $10,000. How much should the builder be required to pay to the promoter in damages? A) $10,000, representing the disco's lost profits. B) $30,000, representing the disco's lost income. C) $300,000, representing damages provided in the contract. D) $310,000, representing damages provided in the contract, plus lost profits.

A) $10,000, representing the disco's lost profits. The promoter will be able to recover $10,000, the disco's lost profits for the 30-day period. The purpose of contract damages is to put the nonbreaching party into as good a position as he would have been in had the breaching party fully performed. This would be represented here by the disco's lost profits, which is income ($30,000) minus expenses ($20,000), or $10,000. (B) is incorrect because giving the promoter the $30,000 income would put him in a better position than he would have been in if the builder had performed, because such an award does not take into account the expenses that were saved by the disco not being in operation. (C) is incorrect because a court would not uphold the liquidated damages clause here. A liquidated damages clause is enforceable only if damages were difficult to estimate at the time the contract was formed, and the amount agreed upon is a reasonable forecast of the damages that would result from a breach. Here, the damages may have been difficult to predict when the contract was formed because it was not known just how well a new discotheque would do; however, the amount agreed upon seems to be too high. The facts indicate that a reasonable estimate at the time of the contract was about $1,000 per day, but because the liquidated damages amount ($10,000 per day) is 10 times that amount, it seems that this amount was unreasonable. (D) is incorrect because it seeks to combine the actual damages with the liquidated damages. Even if the liquidated damages clause were enforceable, this answer would still be incorrect because a party may recover either liquidated damages, or, if not available, the actual damages, but not both.

In March, a homeowner contracted with a buyer to sell his house for $280,000, with the purchase price to be paid and the deed to be delivered on July 1. On May 1, the buyer wrote the homeowner a letter, stating that she has had second thoughts about buying the house, and she "won't pay that amount of money unless you repaint the house and fix up the yard."If the homeowner wishes to treat the contract as breached, which of the following is true? A) He may sue the buyer upon receipt of the letter. B) He must wait until July 1 to sue the buyer, the date on which the purchase price is to be paid. C) He must make a written demand to the buyer seeking adequate assurance of performance and wait a reasonable time for a response prior to filing suit. D) He cannot sue the buyer because the parties' promises are executory at this stage

A) He may sue the buyer upon receipt of the letter. The homeowner may sue the buyer when he receives the letter because an anticipatory breach situation exists. Anticipatory repudiation occurs where a promisor, prior to the time set for performance of her promise, indicates that she will not perform when the time comes. Anticipatory repudiation serves to excuse conditions if: (i) there is an executory bilateral contract with executory duties on both sides; and (ii) the words or conduct of the promisor unequivocally indicates that she cannot or will not perform when the time comes. The requirements for anticipatory repudiation are met here because the homeowner's duty to deliver the deed and the buyer's duty to pay have yet to be performed, and the buyer's writing unequivocally states that she will not pay unless the homeowner performs extra tasks. In the case of anticipatory repudiation, the nonrepudiating party has the option to treat the contract as being breached and sue immediately. Therefore, the homeowner may sue the buyer upon receipt of the letter. (B) is incorrect because, as stated above, the homeowner need not wait until July 1 to sue because the buyer's letter amounts to anticipatory repudiation. The nonrepudiating party may treat this as a total repudiation and sue immediately. The doctrine of anticipatory repudiation would not apply if both sides did not have executory duties to perform. In such a case, the nonrepudiator must wait to sue until the time originally set for performance by the repudiating party. However, as discussed above, the homeowner's duty was still executory at the moment of the buyer's repudiation. Therefore, the homeowner does not have to wait until July 1 to sue the buyer for breach. (C) is incorrect because seeking adequate assurances of the buyer's intent to perform is not necessary when the repudiating party has stated unequivocally that she will not perform. A party may in writing demand adequate assurance of due performance if "reasonable grounds for insecurity arise with respect to the performance of" the other party, after which the nonrepudiating party can treat the contract as repudiated if those assurances are not given within a reasonable time. However, here, it is clear from the buyer's writing that she is unwilling to perform and, therefore, the homeowner need not seek assurances of an intent to perform. (D) is incorrect because the homeowner can sue the buyer immediately because their promises are still executory. As discussed above, the doctrine of anticipatory repudiation is applicable only if there are executory duties on both sides. Here, the homeowner's duty to deliver a deed and the buyer's duty to pay are both executory, so the doctrine applies and the homeowner can sue the buyer now.

State owns empty land on which it wants to create a playground. State and Ursula form a contract under which Ursula is to build one. She is to (a) restyle the land in order that it be suited to a playground and (b) purchase all relevant recreational equipment and place/install it on the land. When Ursula completes the work, State is to (a) reimburse her for all costs she incurs in purchasing recreational equipment and (b) pay her an additional $200,000 fee. Ursula is to finish the project within six months. She begins work. After two months pass, she has spent $20,000 purchasing some of the necessary recreational equipment. She has restyled about one-half of the land. Ursula then runs out of funds and cannot finish the project. She can complete no more of it and so advises State. Although Ursula has breached her contract, State realizes it has suffered no damage but, instead, has come out "ahead." It has paid Ursula nothing. Furthermore, sitting now on its land is part of a playground. For some amount of money—less than $200,000—it can engage another party to complete the work. Hence, State brings no suit against Ursula. Notwithstanding that she has reached the contract, Ursula sues State. Ursula is entitled to recover: $20,000 for quantum valebant (the value of goods and materials furnished) and some amount for quantum meruit (the fair market value of time and effort the plaintiff devotes to performance). $200,000 for realization of her expectation interest. $20,000 for realization of her reliance interest. Which statement is correct? Question 7 options: A) I only B) I and II only C) II and III only D) III only

A) I only Ursula has breached the contract is not able to collect contract damages. Yet, as the party in breach, to prevent unjust enrichment by State, she is able to get restitution on an action for quantum meruit. Generally, the measure of restitution is the value of the benefit conferred. Here, Ursula is entitled to recover $20,000 for the value of goods and materials furnished and some amount for the fair market value of time and effort she devoted to performance until she breached.

A contractor with a contract to deepen a well in a drought-stricken area mistakenly entered onto the wrong property and proceeded to deepen the well there. The owner of the property saw the contractor at work but said nothing. When the contractor completed the job, the property owner refused to pay his bill, and the contractor filed suit. In her answer, the property owner stated that she thought the contractor was employed by the county and that the government was paying for the work because of the drought. She knew, however, that two of her neighbors had recently paid private contractors to deepen their wells.Which of the following arguments offers the contractor his best chance for winning his lawsuit? A) Implied-in-fact contract. B) Promissory estoppel. C) Mutual mistake. D) Unilateral mistake.

A) Implied-in-fact contract. The contractor's best (and only) argument would be that the property owner's silence while the contractor deepened her well was an acceptance by silence of an implied-in-fact contract. An implied-in-fact contract is formed by manifestations of assent other than oral or written language, i.e., by conduct. Where a person knowingly accepts offered benefits, such conduct, viewed objectively, may be said to manifest an agreement to the conferral of such benefits, resulting in a contract implied in fact. While generally an acceptance must be communicated to an offeror to be effective, courts will often find an acceptance where an offeree silently accepts offered benefits. Here, the property owner's purported belief that the work was being done by a county employee at no charge is not plausible. The property owner had no prior notification from or contact with any county employee, and it is not reasonable to believe that anyone, including an employee of the government, would enter upon and disturb private property without prior consent. Moreover, the property owner knew that her neighbors had not had their wells deepened by county workers but had paid private contractors to do the work. Thus, the facts strongly suggest that the property owner's silence as she watched the contractor deepen her well was acceptance by silence of an implied-in-fact contract. (B) is incorrect because promissory estoppel is inappropriate in this case. Under the doctrine of promissory estoppel, as outlined in section 90 of the First Restatement, a promise is enforceable to the extent necessary to prevent injustice if the promisor would reasonably expect to induce action or forbearance of a definite and substantial character and such action or forbearance is in fact induced. Here, the contractor and the property owner whose well he deepened had never met or negotiated for services and had no communications with one another prior to the contractor sending the property owner his bill, and the contractor did not even realize that he was doing the work for that property owner's benefit. Thus, promissory estoppel is not proper. (C) and (D) are incorrect because mistake is a defense to formation of a contract and is raised to render a contract voidable by the adversely affected party. Thus, mistake would not be a ground on which relief could be granted. Even if the property owner's purported mistake were treated as credible by the court, there was no "mutual" mistake because the contractor and the property owner were not mistaken about the samefact, but rather, each was mistaken about a different fact (the contractor, that he was at the right address, and the property owner, that the contractor was a county employee doing the work without charge). Nor could the facts be characterized as a unilateral mistake, in which only one of the parties is mistaken about facts relating to the agreement, because both parties were operating under a mistaken belief. Thus, both (C) and (D) are incorrect.

A nephew, who knew nothing about racing dogs, inherited a greyhound puppy from his uncle. The nephew sold the puppy for $500 to an experienced buyer, who knew of the nephew's ignorance about racing dogs. The buyer began training the dog, but on the first day of training, the dog dropped dead due to a congenital heart defect that was not common to greyhounds and which was discoverable only by autopsy.Under which theory is the buyer most likely to succeed in an action to avoid the contract and get his money back?

A) Mutual mistake Answer A is correct. When both parties entering into a contract are mistaken about existing facts relating to the agreement, the contract may be voidable by the adversely affected party if: the mistake concerns a basic assumption on which the contract is made, the mistake has a material effect on the agreed-upon exchange, and the party seeking avoidance did not assume the risk of the mistake. Here, both parties were mistaken as to the health of the dog and the dog's health was a basic assumption on which the contract was made. The heart defect had a material effect on the exchange; the dog died shortly after the transfer and thus was likely valueless for the buyer's purposes. Finally, nothing in the facts indicates that the buyer bore the risk of the mistake. The defect was uncommon and the only way it could be detected was by an autopsy, so there was nothing the buyer could have done to avoid the risk. Therefore, it is most likely that the buyer will be able to avoid the contract and have his money refunded under a theory of mutual mistake. Answer B is incorrect because both parties were mistaken. If only one of the parties is mistaken about facts relating to the agreement, the mistake will not prevent formation of a contract. However, if the nonmistaken party knew or had reason to know of the mistake made by the other party, the contract is voidable by the mistaken party. As is the case with mutual mistake, for the contract to be voidable, the mistake must have a material effect on the agreed upon exchange and the mistaken party must not have borne the risk of the mistake. Here, the nephew did not know of the heart defect and thus did not know that the buyer was mistaken. Therefore, unilateral mistake is the wrong theory to put forward. Answer C is incorrect because, even if there were a breach of warranty, the remedy is not avoidance of the contract and return of money paid. Implied in every contract for sale by a merchant who deals in goods of the kind sold, there is a warranty that the goods are merchantable. This generally means that the goods must at least be fit for the ordinary purposes for which such goods are used or pass without objection in the trade under the contract description. The first problem in applying this warranty is that the nephew is not a merchant who deals in goods of the kind sold (dogs). The nephew inherited the puppy and knows nothing about racing dogs. Thus, the warranty of merchantability would not be implied in this contract. Moreover, a breach of warranty results in damages consisting of the difference in the value of the goods as delivered and as warranted. The remedy is not to void the contract and return the money. Answer D is incorrect because this warranty does not apply, and the remedy for breach of warranty is damages, not avoidance of the contract. A warranty of fitness for a particular purpose will be implied in a contract for the sale of goods whenever any seller, merchant or not, has reason to know the particular purpose for which the goods are to be used and that the buyer is relying on the seller's skill and judgment to select suitable goods; and the buyer in fact relies on the seller's skill or judgment. Here, there is no indication that the nephew knew the purpose for which the buyer was buying the dog. The buyer was an experienced dog buyer and knew that the nephew knew nothing about dogs. Thus, the buyer was not relying on the seller's skill and judgment, and this warranty would not be implied. Moreover, the remedy for breach warranty is damages, not avoidance of the contract.

A steelmaker purchased a tube rolling machine from a manufacturer of heavy machinery. The machine was sold unassembled for a price of $150,000, with $25,000 payable on delivery and the balance ($125,000) to be paid in 10 monthly installments of $12,500 each. After the machine parts were delivered, the steelmaker contacted an assembly company that specialized in assembly and installation of large and complex manufacturing machinery, and told the company that the machinery had to be up and running within 45 days, or the steelmaker would be in breach of a major contract that it relied on for much of its current revenue. The company agreed, in a written contract with the steelmaker, to assemble and install the tube rolling machine within 45 days at a price of $15,000.Two weeks later, the manufacturer that sold the tube rolling machine to the steelmaker learned that the assembly company was planning to stop work, due to a strike by its labor union. The manufacturer orally offered the assembly company a $3,500 bonus if it would agree to finish the job for the steelmaker. The company accepted the manufacturer's promise and completed the assembly and installation of the tube rolling machine with supervisory personnel within the 45-day time limit set in the agreement between the company and the steelmaker. However, the manufacturer refused to pay the assembly company the $3,500 bonus, so the company sued the manufacturer.Which of the following would be the assembly company's strongest argument to prevail? A) The assembly company owed the manufacturer no preexisting duty to complete the job for the steelmaker, and such completion was sufficient bargained-for consideration for the manufacturer's promise to pay the additional $3,500. B) Because the $3,500 payment was characterized as a "bonus," no further consideration was required and the manufacturer is bound to its promise. C) The assembly company would not have completed the job for the steelmaker within the time limit except in reliance on the manufacturer's promise to pay the additional $3,500. D) By completing the job for the steelmaker, the assembly company conferred a benefit on the manufacturer worth at least $3,500, because such performance assured the steelmaker's ability to pay the manufacturer the balance on the installment purchase agreement for the tube rolling machine.

A) The assembly company owed the manufacturer no preexisting duty to complete the job for the steelmaker, and such completion was sufficient bargained-for consideration for the manufacturer's promise to pay the additional $3,500. The assembly company's best argument is that it owed the manufacturer no preexisting duty to complete the job, and such completion was sufficient bargained-for consideration. Generally, a promise is unenforceable unless it is supported by consideration; thus, for the manufacturer's promise to be enforceable, there must be consideration supporting it. Consideration is defined as a bargained-for exchange of something of legal value. Most courts hold that the thing exchanged will have legal value if it causes the promisee to incur a detriment. A minority of courts hold that a benefit to the promisor is also sufficient. Thus, the company's best argument would be one that includes the idea that it incurred a bargained-for detriment, and this is reflected by (A). The problem with (A) is the preexisting legal duty rule. Traditionally, courts have held that performance of an existing legal duty is not sufficient consideration. However, the rule is riddled with exceptions, and one exception recognized in most jurisdictions applies when, as here, the preexisting duty is owed to someone other than the promisor. Thus, (A) is the best argument because it provides for a full contract recovery. (D) is wrong because it merely reflects the fact that the manufacturer received a benefit. As indicated above, it is the presence of consideration—defined as a bargained-for exchange of something of legal value—that permits the contract to be fully enforced. (A) is a better answer than (D) because it more clearly reflects the basis for finding consideration here. (B) is wrong because merely identifying a promise to pay as a "bonus" does not obviate the need for consideration. For a promise to be enforceable, there must be consideration. (C) is wrong because mere reliance on a promise is not enough to make a contract enforceable. For reliance to provide a substitute for consideration, under the doctrine of promissory estoppel, the promisor must reasonably expect that its promise will induce reliance, and such reliance must reasonably be induced. However, the promise will be enforceable only to the extent necessary to prevent injustice. Here, because the company had a duty to complete the work even without the manufacturer's promise, there is no indication that justice would require payment of the $3,500; there is nothing in the facts to show the company incurred more costs, etc. Thus, the recovery to the company under a promissory estoppel theory would undoubtedly be less than the contract recovery possible under (A).

A homeowner was postponing shoveling the snow off her driveway one Saturday morning because it was so cold outside. When she heard a scream and ran outside, she found the neighborhood newspaper delivery boy injured, lying by his bicycle in her driveway. Feeling terribly guilty, she took the boy to a local doctor and wrote a note to the doctor stating that, if the doctor would treat the boy, the homeowner would cover all of the resulting costs.The boy later admitted that he was attempting to ride the bicycle with his eyes closed, and that his fall had nothing to do with the snow in the homeowner's driveway. After treating the boy for his injuries until he recovered, the doctor sent the homeowner a bill for his services. When the homeowner refused to pay, the doctor brought an action to recover the amount of his bill.In the doctor's suit against the homeowner, which party is likely to prevail? A) The doctor, because he gave medical treatment to the boy after receiving the homeowner's note. B) The doctor, because the homeowner's promise to pay the boy's medical expenses was in writing. C) The homeowner, because there was no consideration for her promise to the doctor. D) The homeowner, because she derived no benefit from the medical services rendered to the boy.

A) The doctor, because he gave medical treatment to the boy after receiving the homeowner's note. The doctor is likely to prevail because he performed his duties under their contract and is entitled to the homeowner's performance. The homeowner offered to pay the doctor if he treated the injured newspaper delivery boy. The doctor accepted by treating the boy; this treatment was also his consideration for the contract. Thus, the homeowner was contractually bound to pay for the medical expenses and (A) is correct. (B) is incorrect because the homeowner's promise need not have been in writing to be enforceable. The requirement of a writing under the Statute of Frauds for promises to pay the debt of another is not applicable here. The homeowner's promise was not a suretyship promise that was collateral to a promise of the boy; it was an independent promise by the homeowner to be primarily liable for the boy's medical expenses, and therefore need not be in writing to be enforceable. (C) is incorrect because the doctor's treatment is the consideration for the homeowner's promise to pay. It does not matter that the boy, rather than the homeowner, received the benefit of the doctor's services. Services provided to the newspaper boy was the legal value that the homeowner bargained for. (D) is incorrect because the doctor's treatment was the benefit that the homeowner sought to derive from her bargain. As discussed above, it does not matter that the boy, rather than the homeowner, benefited from the doctor's services. That was what the homeowner bargained for, and she received the benefit of her bargain. Also, remember that the benefit need not have economic value; the homeowner's benefit here is her peace of mind.

A buyer for a toy store emailed a toy distributor to place an order for 1,000 units of a video game. In the email, the buyer asked the distributor if the game was compatible with a certain popular video gaming system since the games were being ordered for resale. The distributor's reply said, "That gaming system's name is listed on the packaging of the game." Both parties were unaware that the video game company had just released a new generation of consoles. Upon receiving the shipment, the buyer discovered that the game does not work with the latest console. The buyer is uncertain whether it can re-sell all of the games since they only work on older consoles.Which of the following is the most appropriate remedy for the buyer? A) Specific performance of the contract, requiring the manufacturer to ship video games compatible with all generations of consoles. B) Rescission of the contract. C) A refund on any games that the buyer is unable to re-sell. D) The difference between the purchase price in the contract and the value of the games as received.

B) Rescission of the contract. The buyer is entitled to rescission of the contract on the ground of mutual mistake. Rescission is a remedy that discharges the contractual duties of the parties and puts an end to the transaction, leaving the parties as though the contract had never been made. Unilateral rescission results where one of the parties desires to rescind but the other party refuses to agree to a rescission. For unilateral rescission to be granted, the party desiring rescission must have adequate legal grounds. One ground for unilateral rescission is mutual mistake of a material fact. If the distributor reasonably believed that the games were compatible with the named gaming system when shipped to the buyer, then both parties were operating under a mistake of fact. This mistake concerns a basic assumption on which the contract is made, because the compatibility of the game with that gaming system was a fact without which the buyer would not have entered into the transaction. The mistake creates a material imbalance in the agreed exchange, and the party seeking avoidance (the buyer) did not assume the risk of the mistake because she asked if the games were compatible at the time of placing the order. Because of this mutual mistake of material fact, which existed at the time the contract was entered into, the buyer is entitled to rescission of the contract. (C) is not a standard remedy for a breach of contract. (D) only offers the buyer monetary damages, which may be minimal. The value of the game for resale may be the same provided that the buyer is able to re-sell the game to a consumer, but here the buyer is uncertain whether it can re-sell this quantity of games. (A) is incorrect because a buyer has a right to specific performance of a sales contract only where the goods are unique or in other proper circumstances. [UCC §2-716] There is no indication that this video game is unique. Thus, a contract for the sale of such goods is not an appropriate subject for specific performance.

A landowner entered into a written agreement with a real estate broker whereby the broker would receive a commission of 10% of the sale price if he procured a "ready, willing, and able buyer" for the landowner's property and if the sale actually proceeded through closing. The broker found a buyer who agreed in writing to buy the property from the landowner for $100,000, the landowner's asking price. The buyer put up $6,000 as a down payment. The agreement between the landowner and the buyer contained a liquidated damages clause providing that, if the buyer defaulted by failing to tender the balance due of $94,000 at the closing date, damages would be 10% of the purchase price. The landowner included that clause because she was counting on using the proceeds of the sale for a business venture that would likely net her at least $10,000. The buyer became seriously ill and defaulted. When he recovered, he demanded that the landowner return his $6,000, and the landowner refused. The broker also demanded the $6,000 from the landowner and was refused. The broker and the buyer filed separate suits against the landowner, with the buyer pleading impossibility of performance. The two cases are consolidated into a single case. How should the court rule as to the disposition of the $6,000? A) The landowner keeps the entire $6,000, because the liquidated damages clause is reasonable. B) The buyer gets the entire $6,000, because his performance was impossible. C) The broker gets the entire $6,000, which is 60% of the commission he is entitled to, because he substantially performed his part of the contract by producing a buyer willing to pay the $100,000 asking price. D) The broker gets $600 and the landowner gets $5,400, because the damages clause was reasonable and the broker is entitled to 10% of whatever the landowner realizes from the deal.

A) The landowner keeps the entire $6,000, because the liquidated damages clause is reasonable. The landowner may keep the $6,000 as liquidated damages. A liquidated damages clause is enforceable if: (i) damages are difficult to ascertain at the time of the making of the contract, and (ii) the damages are a reasonable forecast of compensatory damages. Here, the landowner was unsure what her damages would be if she did not receive the sales proceeds from the property, but $10,000 seemed a reasonable amount. Thus, both criteria for valid liquidated damages clauses are met. (B) is incorrect because impossibility must be objective; i.e., performance cannot be accomplished by anyone. Physical incapacity of a person necessary to effectuate the contract may discharge contractual duties if that person's performance is clearly impossible. (Usually this occurs in personal services contracts, where only that one person can perform the required duty.) Although the buyer was seriously ill, it is not clear that this made it impossible for him to produce the $94,000. Without more facts, it is reasonable to assume that someone else could have delivered the money or that his mortgage would still have gone through, etc. (C) is incorrect because the conditions for the broker's payment were not met: It is debatable whether he produced a "ready, willing, and able" buyer, and in any event the sale did not actually proceed through closing. (D) is incorrect because the broker was to receive proceeds from the sale of the property; the $6,000 was damages and not sale proceeds.

A bulk retailer of accessories for musical instruments placed an advertisement in a trade magazine popular with those in the music business, offering for sale 50-count boxes of a particular type of mouthpiece for use with the French horn, minimum purchase 10 boxes, at $100 per box. In response to the advertisement, the owner of a large store that sold brass and woodwind instruments in its shop and over the Internet sent a written order to the bulk retailer for 12 boxes (50-count) of the mouthpiece. In his letter that accompanied the order, the store owner stated that he would send the bulk retailer his payment of $1,200 upon delivery. The letter also said that the mouthpieces must fit onto three specified models of French horn.The day after receiving the written order and letter from the store owner, the bulk retailer shipped 12 boxes (50-count) of the mouthpiece to him. Accompanying the invoice on the boxes was a letter from the bulk retailer stating that the mouthpieces are compatible with two of the models of French horn, but that the retailer makes no warranties as to the compatibility of the mouthpieces with any other model of French horn. Shortly after accepting shipment of the boxes, the store owner realized that the mouthpieces did not fit onto the third model of French horn that it had specified and instituted an action against the bulk retailer.Which of the following statements would offer the strongest support in favor of the store owner's position? A) The store owner's letter was an offer, and shipment of the units was an acceptance. B) The store owner's letter was an offer, and the bulk retailer's letter accompanying the invoice was an acceptance. C) The bulk retailer's letter was an offer, and acceptance of the units by the store owner was an acceptance of the offer. D) Shipment of the units was a counteroffer, and acceptance of the units by the store owner was an acceptance of the counteroffer.

A) The store owner's letter was an offer, and shipment of the units was an acceptance. The store owner's best position is that his letter was an offer, and shipment of the mouthpieces was an acceptance; thus, the shipment of nonconforming goods both created a contract and a breach of that contract, affording the store owner an immediate cause of action. The contract at issue involves the sale of goods, and is thus governed by Article 2 of the UCC. Under UCC section 2-206, an offer to buy goods for current or prompt shipment is construed as inviting acceptance either by a promise to ship or by current or prompt shipment of conforming or nonconforming goods. The shipment of goods, even though they are nonconforming, is an acceptance creating a bilateral contract and a breach unless the seller reasonably notifies the buyer that the nonconforming goods are offered only as an accommodation to the buyer. Here, the store owner's letter was an offer that invited the bulk retailer to accept by either a promise to ship or a prompt shipment. The bulk retailer's shipment of the mouthpieces is an acceptance of the store owner's offer, because the bulk retailer's letter accompanying the shipment was probably not sufficient as an accommodation notice (which would have made the shipment a counteroffer rather than an acceptance). The shipment contains nonconforming goods, because the mouthpieces are not compatible with the third type of French horn specified in the store owner's offer. The shipment of nonconforming goods as an acceptance both created a bilateral contract between the parties and constituted a breach of that contract by the bulk retailer, thus allowing the store owner to sue for any appropriate damages for breach of contract. (B) is incorrect because, since the bulk retailer's letter was an acceptance, there is a chance that the store owner will be bound by the terms of the letter. Between merchants, additional proposed terms in an acceptance become part of the contract unless: (i) they materially alter the original contract; (ii) the offer expressly limits acceptance to its terms; or (iii) the offeror has already objected to the particular terms, or objects within a reasonable time after receiving notice of them. [UCC §2-207] The bulk retailer and the store owner are both merchants, in that they deal in goods of the kind involved in the contract. Insofar as the bulk retailer's letter is an acceptance of the store owner's offer, the sentence disclaiming compatibility of the mouthpieces with other models of French horns is an additional proposed term. The store owner's offer did not expressly limit acceptance to its terms, nor did the store owner object to the additional terms. Thus, the additional terms will become part of the contract unless the court finds that they materially alter the original contract. If these terms are held to be part of the contract, the store owner will have no cause of action for the incompatibility of the mouthpieces with one of the French horn models. Because (B) creates a framework in which the store owner might lose (i.e., if it is found that the additional terms do not materially alter the original contract), (B) is not as good an answer as (A). (C) and (D) represent the worst scenarios for the store owner. If the bulk retailer's letter was an offer as stated in (C) and acceptance of the mouthpieces by the store owner was an acceptance of the offer, there is a contract based on the terms in the bulk retailer's letter; i.e., no warranties as to the compatibility of the mouthpieces with any other model of French horn. In that event, the store owner would have no cause of action against the bulk retailer based on the mouthpieces' incompatibility with one of the French horn models. The same result would arise if shipment of the mouthpieces were treated as a counteroffer as stated in (D) (this would be the result if the bulk retailer's letter were held to be a sufficient accommodation notice under UCC section 2-206(1)(b)). Acceptance of the mouthpieces would be acceptance of the counteroffer according to the terms of the letter accompanying the shipment, so that the store owner could not bring an action based on the mouthpieces' incompatibility with one of the French horn models.

A downtown department store engaged an electrician to service all electrical appliances sold by the store for a flat fee of $5,000 per month. Under a written contract signed by both parties, the store was responsible for pickup and delivery of the appliances to be repaired and the billing for the work. By its terms, the contract would continue until either party gave 180 days' written notice of its intent to terminate. Several months ago the electrician informed the store that he was losing money on the deal and was in financial trouble. He requested in good faith that the fee for the next three months be increased by $1,000 and that this increase be paid to a local bank to help pay off a loan that the bank had made to the electrician. The store orally agreed to so modify the original contract. However, the store did not pay the bank and now the bank is suing the store for $3,000.Who will prevail? A) The store, because there was no consideration to support the promise to pay the bank. B) The store, because the bank is only an incidental beneficiary of the modified contract between the store and the electrician. C) The bank, because it is an intended creditor beneficiary of the modified contract between the store and the electrician. D) The bank, because the electrician exercised good faith in requesting the modification regarding the payment to the bank.

A) The store, because there was no consideration to support the promise to pay the bank. The store will prevail, because there was no consideration to support its promise to pay the bank the additional $1,000 per month. This question looks like it concerns third-party beneficiaries, but it actually presents a consideration issue. Generally, there must be consideration for modification of a contract, and a promise to perform an act that a party is already obliged to do is not sufficient consideration (the "preexisting legal duty" rule). Here, the electrician is promising to do exactly what he was obliged to do under his original contract with the store; thus, there is no consideration to support the promise to increase the fee. Note that the modern view permits modification without consideration if it is fair and equitable in view of unanticipated circumstances. That is not applicable here. This exception contemplates an unanticipated circumstance arising in performance of the contract that makes performance more difficult or expensive. (B) is wrong because the bank is an intended beneficiary, not an incidental beneficiary. An intended beneficiary is one who is clearly intended to benefit from the agreement. Here, the bank was named in the agreement and performance was to be made directly to it, and so it is clearly an intended beneficiary. (C) is wrong even though it is true that the bank is an intended creditor beneficiary. Despite this status, the bank will not recover because there was no consideration to support the modification of the contract. The status of creditor beneficiary does not give the bank any more rights than the electrician would have had to enforce the agreement, and the electrician could not enforce the agreement for the additional money because there was no consideration. (D) is wrong because it is based on the rule of UCC section 2-209, which states that an agreement subject to the UCC does not need consideration to be binding. However, the UCC governs only in cases of the sale of goods, and this question presents a contract for services. Thus, the UCC does not apply and the common law rule requiring consideration controls.

A dairy farmer hired a local company to assemble milking machines that the farmer had purchased. The written contract between the parties provided that the company would assemble and install the milking machines in the farmer's dairy barn within 30 days, in time for the arrival of some additional cows, and the farmer agreed to pay the company $10,000.Three weeks into the job, the company realized that it would lose $2,500 on the job, due to a new wage agreement forced on the company by its employees' union after the contract was executed. The company approached the farmer and told him that the job could not be completed for less than $12,500.After some discussion, the farmer and the company executed an agreement obligating the farmer to pay an additional $2,500 upon completion of the job. The company completed the work on time, but the farmer now refuses to pay the additional $2,500.In a suit by the company against the farmer, which of the following would be the farmer's best defense? A) This was a contract for services and the modification was not supported by consideration. B) The modification is voidable because the company knew that the farmer needed the machines up and running in 30 days and took advantage of his duress. C) A contract cannot be modified once performance has begun, and the attempted modification occurred three weeks into the job. D) During initial contract negotiations, the company assured the farmer that the milking machines would be assembled and installed for no more than $10,000.

A) This was a contract for services and the modification was not supported by consideration The farmer's strongest position is that he came under no duty to pay the additional $2,500 because this was a contract for services rather than goods. The general rule is that a modification of a contract for services must be supported by consideration. Because a preexisting legal duty is not consideration, for a modification to be enforceable there must be a new promise made by each side. Here, the farmer agreed to pay an additional $2,500, but the company promised nothing in return. Thus, the modification is unenforceable. Note that the UCC enforces a good faith modification even without consideration, but the UCC applies only in the sale of goods, and the contract here was for services (assembly and installation of the milking machines). (B) is an arguable position, but the majority of jurisdictions would not find duress under the facts here, where one party is merely taking economic advantage of the other party's pressing need to enter into the contract. (C) misstates the law. A contract can be modified at anytime before performance is complete. (D) is not the best position because what was discussed at the negotiations is irrelevant; the parol evidence rule would prohibit introduction of such statements (when there is an integrated writing, prior and contemporaneous oral statements are inadmissible to vary its terms).

On April 15, a wholesaler of tulip bulbs telephoned a local nursery and offered to sell to the nursery 80 gross of tulip bulbs for $8,000, not including delivery charges. The nursery accepted immediately. On April 17, the nursery sent the wholesaler an email confirming the deal for the sale of 80 gross of tulip bulbs for $8,000, and stating that it anticipated a waiver of the delivery charges because of the size of the order. On May 3, the wholesaler telephoned the nursery and stated that, due to a poor growing season for tulips, it would not be able to supply any tulip bulbs to the nursery.If the nursery brings suit against the wholesaler and the wholesaler asserts the Statute of Frauds as a defense, will the nursery prevail? A) Yes, because its April 17 email contained the quantity term. B) Yes, because its April 17 email contained the price term. C) No, because the nursery's April 17 email varied the terms of the wholesaler's offer. D) No, because the wholesaler is the party to be charged and has signed nothing.

A) Yes, because its April 17 email contained the quantity term. Because the quantity was stated in the April 17 email, the Statute of Frauds is satisfied and the nursery may prevail. This contract is for the purchase and sale of goods; thus, the UCC applies. The Statute of Frauds requires that a contract for the sale of goods for $500 or more be evidenced by a writing signed by the party to be charged. This writing must contain the essential elements of the agreement. The quantity term is the key to the sufficiency of a memorandum, and here the writing includes the quantity term. Thus, the writing complied with the Statute of Frauds. (B) is wrong because all other terms (including price) may be proved by parol evidence. The UCC requires only that the memorandum contain (i) quantity, (ii) the signature of the party to be charged, and (iii) a writing sufficient to indicate that a contract was formed. [UCC §2-201] (C) is wrong because it does not bear on the Statute of Frauds issue, but rather on the issue of the additional terms, which will not prevent a contract from being formed between merchants. (D) is wrong because UCC section 2-201(2) provides that, in a deal between merchants, a writing confirming the deal sent by one party will bind both parties, unless the other party objects in writing within 10 days. Here, the wholesaler did not object within 10 days, and so the nursery's email confirmed the deal. Thus, the wholesaler can be charged even though the wholesaler has not signed the memorandum.

A corporation whose subsidiaries include a major hotel chain planned to build a new hotel and advertised for bids to build the hotel within the next six months. Four bids were received, for $17 million, $17.2 million, $17.4 million, and $15 million. The corporation's chief financial officer reviewed the bids, then emphatically told the corporation's chief executive officer ("CEO") that there was "no way" the low bidder could make a profit on the $15 million bid. The CEO made no response.In fact, the builder had stayed up for 72 hours without sleep preparing the bid for the hotel project and had neglected to include the plumbing expenses in the bid. Typically, the cost of plumbing, including the shop's profit, would have been about $2 million.Shortly after the $15 million contract was signed by the CEO and the builder, the builder discovered his mistake and telephoned the CEO to tell her that he had forgotten to include the cost of plumbing, adding that he would normally charge $2 million for plumbing. The CEO agreed to pay the additional $2 million, but this arrangement was never reduced to writing. After the builder completed the project on time, the CEO sent him a check for only $15 million.Can the builder compel the CEO to tender the additional $2 million? A) Yes, because the CEO was on notice of the builder's mistake. B) Yes, because the builder relied to his detriment on the CEO's promise. C) No, because the builder had a preexisting legal duty to complete the project for $15 million. D) No, because evidence of the agreement to pay the additional $2 million is barred by the parol evidence rule.

A) Yes, because the CEO was on notice of the builder's mistake. The builder will be able to compel the CEO to pay the additional $2 million because the CEO was on notice of the mistake. The builder has the defense of unilateral mistake. Although the general rule is that a contract will not be avoided by a unilateral mistake, there is an exception where the nonmistaken party either knew or should have known of the mistake. Here the facts clearly indicate that the CEO knew that the builder's bid could not be correct, yet relied on it anyway. Thus, the builder had grounds to avoid the contract. Rather than completely avoid the contract here, the parties agreed to reform it, but they failed to record the reformation in writing. Nevertheless, the court will allow the parties to show the reformed terms because of the mistake. (B) is incorrect because the fact that the builder relied to his detriment on the CEO's promise would not allow him to collect the additional $2 million. Detrimental reliance is a factor in promissory estoppel, which is a doctrine used by the courts to avoid an unjust outcome when there is no contract. Here, there is a contract supported by consideration, so this doctrine does not apply. The builder is entitled to reformation of the bargained-for contract because the CEO was on notice of the builder's mistake. In addition, even if this were a proper case for promissory estoppel, the builder would not necessarily be entitled to the $2 million. Courts that follow the Second Restatement approach typically award reliance, not expectation, damages. (C) is incorrect because the unilateral mistake here was sufficient to discharge the builder from his duties under the contract, so there was no preexisting duty. (If the mistake had not been sufficient to discharge the builder, (C) would be correct because where one is under a preexisting legal duty to perform, performance of that same duty generally will not be sufficient consideration to support a promise to pay additional sums for the performance.) (D) is incorrect because the parol evidence rule only prevents introduction of oral statements made prior to or contemporaneously with a written contract. Here, the $2 million term, although oral, was agreed upon after the original contract was made; thus, the parol evidence rule would not be a bar.

A farmer in a small town suffering a severe drought contacted a scientist in another county who claimed to have perfected a rainmaking machine. The farmer and the scientist entered into an agreement providing that the scientist would be paid if he made it rain. The agreement did not specify an amount of payment or any deadline by which it must rain. After several days of trying without success, the scientist said that the machine might work better at a higher elevation. A neighbor who lived on higher ground agreed to let the scientist place the rainmaking machine on his land and further told the scientist that if he made it rain before the end of the following day, he would pay him $20,000. The scientist placed the machine on the neighbor's land. By nightfall of the same day, clouds had begun to gather over the town, and the next morning it was pouring rain.If the neighbor refuses to pay the $20,000 to the scientist, can the latter enforce the promise to pay? A) Yes, because the neighbor's promise was supported by the consideration that the scientist make it rain by the next night. B) Yes, but only in the amount of the reasonable value of his services. C) No, because the scientist had a preexisting duty to make it rain as a result of the agreement with the farmer. D) No, because the neighbor's promise constituted no legal detriment to him.

A) Yes, because the neighbor's promise was supported by the consideration that the scientist make it rain by the next night. The scientist can enforce the contract with the neighbor because it was supported by consideration. For a contract to be enforceable, there must be an offer, an acceptance, and consideration. Here, there was an offer made by the neighbor (i.e., if the scientist made it rain before the end of the following day, he would pay the scientist $20,000), the scientist accepted the offer by performing, and the performance was valid consideration because the scientist was not under a preexisting duty to make it rain by the next evening (the scientist agreed with the farmer to try to make it rain, but there was no time limitation and no specified consideration). Thus, the neighbor's condition that the scientist had to make it rain by the following day was new consideration and not a preexisting duty. Therefore, the contract is enforceable. (B) is incorrect because a contract is usually governed by the terms agreed on by the parties, and the neighbor and the scientist agreed that if the scientist made it rain by the next day, the neighbor would pay the scientist $20,000. Thus, the scientist should be paid $20,000. (C) is incorrect because, as discussed above, the scientist's making it rain by the next day was consideration sufficient to support the neighbor's promise to the scientist. Moreover, the scientist probably did not owe any duty to the farmer in the first place, because the offer appears to be for a unilateral contract—an offer looking for performance rather than a promise to perform, which does not become binding until performance is rendered. (D) is incorrect because a promise to pay $20,000 in exchange for performance is a legal detriment.

On December 15, a law firm received an offer from an office supply superstore consisting of its catalog and a signed letter stating, "We will supply you with as many of the items in the enclosed catalog as you order during the next calendar year. We assure you that this offer and the prices in the catalog will remain firm throughout the coming year." No other correspondence passed between the office supply superstore and the law firm until the following April 15, when the superstore received an e-mailed order for 100 reams of premium paper, a catalog item, from the law firm. The catalog lists one ream of premium paper at $4, but due to a worldwide shortage, on April 15, the superstore's price for a ream of premium paper was $6. Is the superstore obligated to supply the law firm with the premium paper at $4 per ream? A) Yes, because the superstore had not revoked its offer. B) Yes, because a one-year option contract was created by the superstore's offer. C) No, because under applicable law, the irrevocability of the superstore's offer was limited to a period of three months. D) No, because the law firm did not accept the superstore's offer within a reasonable time.

A) Yes, because the superstore had not revoked its offer. Answer A is correct. An offer may be accepted at any time before the offer is terminated (e.g., by revocation or lapse of time). The offeree must accept the offer within the time period specified. Here, the superstore's offer gave the law firm one year to accept. Because it had not revoked the offer, the superstore's offer was still effective on April 15 when the law firm e-mailed an acceptance. The firm offer, made in writing and signed by a merchant, was effective from January 1 for a period of one year, and under the UCC, it was irrevocable until 90 days thereafter. (See discussion in Answer C, below.) As of April 15, the offer had not been revoked; therefore, the affirmative pledge not to raise prices was still in effect. Thus, the law firm's order constituted an acceptance of the offer. Answer B is incorrect. An option is a distinct contract in which the offeree gives consideration for a promise by the offeror not to revoke an outstanding offer. Here, there was no distinct contract in which the law firm gave consideration to the superstore to hold the offer open. Thus, there was no option contract. Moreover, an option contract would not be necessary because the issue is not whether the superstore could revoke the offer. The superstore never attempted to do so. Answer C is incorrect because it states the rule for a merchant's firm offer, but that does not come into play here. Paper is goods; thus, this is a contract for a sale of goods and Article 2 of the UCC applies. Under Article 2, if a merchant offers to buy or sell goods in a signed writing and the writing gives assurances that it will be held open, the offer is not revocable during the time stated (or a reasonable time if no time is stated) except in no event may the period exceed three months. Here, the superstore is a merchant who provided a signed written offer giving assurances that the offer would be held open for one year. If, as here, a merchant-offeror states that an offer will stay open for a period beyond the UCC's three-month limit on irrevocability, it will be bound only for three months. After that point, the offer becomes revocable. Here, the superstore did not revoke the offer after it became revocable. Thus, the law firm's acceptance was valid and formed a contract on the offered terms. Answer D is incorrect. If no time period is specified in the offer, the offeree must accept within a reasonable time. If it does not do so, then it will have allowed the offer to terminate. Here, reasonable time is irrelevant because the offer was made for a fixed period of time, and it was not revoked prior to the acceptance.

A manufacturer of washing machines telephoned an out-of-state parts supplier and said, "I have an order for 250 washing machines, which require motors made according to the specifications I sent you last week. Are you in a position to quote me a price for making these motors?" The supplier said he would ship them F.O.B. to the manufacturer's factory for $20 each, with delivery in 30 days. The manufacturer told the supplier to enter the order. The supplier then said, "I will send you a statement of the terms we agreed to soon." What is the result of the telephone conversation between the manufacturer and the supplier? A) The manufacturer's first statement was an offer. B) A contract was formed and it will be enforceable once the supplier's confirmatory memo is sent. C) No contract was formed, because there was never any acceptance of the supplier's offer. D) The supplier accepted the manufacturer's offer when he said he would ship them F.O.B. to the manufacturer's factory for $20 each, with delivery in 30 days.

B) A contract was formed and it will be enforceable once the supplier's confirmatory memo is sent. Answer B is correct. The conduct of the parties during this phone conversation clearly indicates that they intended to form a contract. There was an offer to sell the goods for $20 each and acceptance of that offer, and consideration on both sides. However, under Article 2, to be enforceable, a contract for the sale of goods worth at least $500 needs to be in writing, but a merchant's confirmatory memo sent within a reasonable time will be sufficient. Answer A is incorrect. The manufacturer's first statement was not an offer; it was nothing more than an inquiry about price or preliminary negotiations. The more definite a communication is, the more likely it will be considered an offer. The manufacturer's first statement was an inquiry about price only, and there is no indication that this statement alone manifested any intent on the manufacturer's part to purchase motors from the supplier absent a satisfactory price. Although subsequent statements by the parties in this conversation manifested their intent to form a contract, the manufacturer's first statement is not an offer. If anything, the supplier's first statement, that he would ship them to the manufacturer for $20 each, was the original offer. Answer C is incorrect. When the manufacturer told the supplier to enter the order, quite clearly an acceptance occurred. Even under the strict common law rule, a party did not need to use the exact words "offer" or "accept" to form a contract. An acceptance is merely assent by an offeree to a proposal made by an offeror. Answer D is incorrect. There was no offer to accept at the point in time here. The manufacturer had merely inquired into a price. He had, in effect, asked for an offer. The supplier made the initial offer when he stated that he could ship the motors to the manufacturer. The manufacturer's words to enter the order, in response to the supplier's offer, were an acceptance of the offer to sell and ship the goods.

For work as a volunteer, Lanette has won a $10,000 "Civil Society" award, subject to the condition that she attend the Civil Society Award ceremony at 10 AM on June 12 in the town of Hatfield, 200 miles from her home. Others have advised Lanette to take a hotel room in Hatfield for the night of June 11, so she might easily reach the ceremony venue in the morning. But Lanette decides to do otherwise. She plans to drive to Hatfield by automobile, leaving at 6:00 AM on the morning of June 12. Knowing that one of her car's tires has worn its tread, she takes it on the morning of June 11 to Al, an auto mechanic. Orally, she and Al form a contract under which Al will replace the worn tire and she, in exchange, will pay him $150. Late that same day, Lanette contacts Al: "Is my car ready?" Al says, "Yes, come on in and take it." Lanette travels to Al's facility, pays him $150, and drives her car away. As Lanette does not know, and as Al does not know, the tire has not been replaced. By accident of paperwork, Al thought his assistant had changed the tire when, in fact, he had not. On the next morning, while Lanette drives to Hatfield, the worn tire goes flat. For that reason, Lanette is delayed by one hour and reaches Hatfield at 11:00 AM—too late to accept her $10,000 award. Instead, the award goes to the "runner-up." Lanette sues Al alleging that his breach of contract caused her $10,000 in damages. She demands a recovery in that amount. On hearing all evidence adduced at trial, the court issues a directed verdict for Lanette in the amount of $150. Most likely the court's reason for not awarding Lanette $10,000 is that: Question 5 options: A) Lanette cannot reasonably expect an award of $10,000 in exchange for a payment of $150. B) Al had no reason to foresee that a breach on his part could produce so much damage as that. C) Al's failure to replace the tire amounted not to breach of contract, but to innocent error. D) Lanette could have reduced her damage—to $00.00—if she had taken a hotel room in Hatfield as others suggested she do.

B) Al had no reason to foresee that a breach on his part could produce so much damage as that Answer B is correct. $10,000 in damages here (in addition to $150 of direct loss) would be consequential damages. These damages may be recovered only if at the time the contract was made, a reasonable person would have foreseen the damages as a probable result of a breach. Foreseeability is the key issue for consequential damages. To recover consequential damages, the plaintiff must show that the breaching party knew or had reason to know of the special circumstances giving rise to the damages. Because of the foreseeability limitation on damages, the court would not award $10,000 to Lanette if Al had no reason to foresee that a breach on his part could produce so much damage as that.

An aunt agreed to pay for any lace shawl that her niece purchased at a certain local shop if the niece wore the shawl to the town's ethnic festival. The niece despised shawls, but she really loved her aunt and did not want to hurt her feelings, so she purchased a $300 lace shawl from the shop. She accompanied her aunt to the ethnic festival wearing the shawl, and the aunt was very pleased. The aunt died shortly after the festival and her estate refused to reimburse the niece for her purchase. The niece filed suit to collect the $300 from the aunt's estate.Which legal theory will offer the niece her best chance of winning the case? A) Quasi-contract. B) Bargain and exchange. C) Conditional gift. D) Account stated

B) Bargain and exchange. The legal theory of bargain and exchange would offer the niece her best chance of collecting from her aunt's estate. To be enforceable, a contract must be supported by consideration. Generally, consideration requires a bargained-for exchange of something of legal value from each party. To be bargained for, the promise must induce the detriment and the detriment must induce the promise. Here, the niece was induced to purchase the shawl and incur a debt in exchange for her aunt's promise to pay, and the aunt was induced to pay in exchange for her niece's promise to purchase the shawl and attend the festival. It does not matter that the aunt did not receive an economic benefit—influencing her niece's mind is sufficient to establish an exchange. Both parties gave something of legal value because the aunt had no legal duty to pay for a shawl for her niece and the niece had no legal duty to purchase the shawl and attend the festival. Therefore, there was a bargained-for exchange of something of legal value and a contract was formed. (A) is wrong because there was a contract that would be enforceable (see above). Restitution in quasi-contract is available in cases where a contract is unenforceable and one of the parties has performed, resulting in unjust enrichment to the other party, or in cases where there is no contract but the plaintiff has conferred a benefit on the defendant with the reasonable expectation of being paid, the defendant knew or had reason to know of the plaintiff's expectation, and the defendant would be unjustly enriched if she were allowed to retain the benefit without compensating the plaintiff. This theory would not be appropriate because there is an enforceable contract and an adequate legal remedy is available. (C) is wrong because a promise to make a gift is unenforceable for lack of consideration, and this result is not changed where the promise is conditional and the conditions have been fulfilled. In fact, an argument that there was a conditional gift would negate the bargain and exchange needed to establish an enforceable contract, because it would involve arguing that the aunt did not intend to induce her niece's detriment (i.e., the aunt was not seeking to trade her promise to pay in exchange for her niece's promise to attend the festival), but rather was merely stating on what conditions the niece could receive her gift. Thus, (C) would not be helpful to the niece's case. (D) is wrong because the theory of account stated is used in cases where the parties to a contract have had a series of transactions and agree to a final balance due from one to the other. Here, the niece is seeking to enforce a single transaction and, thus, the theory would not be appropriate.

A man fed a lost cat and returned it to its owner. She was so happy to see her cat that she promised to leave the man money in her will. When the cat owner died a few days later, the man visited the owner's daughter demanding his share of the estate. All of the cat owner's bank accounts had been held in joint tenancy with her daughter. The daughter reluctantly signed a document stating that she would pay the man $500 in exchange for his agreement not to sue her mother's estate. She later had second thoughts and refused to pay the man on the ground that there was no consideration for the agreement.Besides the consideration stated in the daughter's written instrument, what other fact would strengthen the man's claim? A) He never would have fed and returned the cat had he known that he would not receive any payment for his efforts. B) He believed he could sue the cat owner's estate. C) The majority of the funds in the cat owner's bank account were royalties from a series of television commercials starring the cat. D) The cat owner's promise to him was in writing.

B) He believed he could sue the cat owner's estate. If the man had a reasonable (i.e., good faith) belief in the enforceability of his claim, his surrender of the claim is valid consideration. (A) is wrong because his motive for feeding and returning the cat is immaterial. (C) is wrong. The source of the cat owner's money is irrelevant. In addition, the accounts were held in joint tenancy and any interest the cat owner may have had in the funds ended when she died. (D) is wrong because there was no consideration given for the cat owner's promise and the fact that it was in writing does not change the lack of consideration.

Two cousins had been raised in the same neighborhood. When the elder cousin married, she moved to another city, but the two cousins corresponded frequently over the years and occasionally visited each other. On March 1, the elder cousin wrote to her younger cousin that her last child had married and moved away. In the letter, she wrote, "I wonder if I should sell my house. It's really too big for me. If I could get $85,000 for it, I'd sell it and move into an apartment." On April 15, the younger cousin replied in a letter: "We just learned that we will be moving to your city. My husband and I both like your house and would be glad to pay you $95,000 for it. We could pay $10,000 in cash, but we would have to get a mortgage for the rest." Upon receiving the letter on April 17, the elder cousin telephoned her younger cousin and said "I accept your wonderful offer. I have a mortgage on the house for $85,000, so you can just pay me $10,000 and take over the payments." The younger cousin responded: "That's great. Let me talk to my husband about it again to make certain that we both agree. You'll hear from me within 10 days." On April 18, a local real estate developer offered the elder cousin $100,000 for her house, and she entered into a written purchase-and-sale agreement with him. An hour later, the younger cousin called and before her cousin could say anything she blurted out, "Great news! I talked with my husband, and we are delighted to accept your offer."Will the younger cousin prevail in a suit against the elder cousin for breach of contract? A) No, because the younger cousin said she had to talk to her husband to make certain they both agree. B) No, because of the Statute of Frauds. C) Yes, because the series of correspondence serves as sufficient written memoranda evidencing a contract. D) Yes, because the younger cousin was unaware of the sale to the real estate developer before calling the elder cousin.

B) No, because of the Statute of Frauds. Answer B is correct. The Statute of Frauds requires a contract related to an interest in land to be evidenced by a written memorandum, signed by the party to be charged. Here, the elder cousin is the party to be charged. Thus, even though there is an agreement between the two cousins (the offer to buy was accepted and there was consideration on both sides), it is not enforceable against the elder cousin, because she did not sign a writing evidencing a contract and can raise the defense of the Statute of Frauds. Answer A is incorrect. An acceptance must be unequivocal. However, at the time the younger cousin said she had to check with her husband, the cousins had already formed a contract. The younger cousin had made an offer that the elder cousin had already accepted. Thus, the younger cousin's comment about her husband had no effect on the formation or enforceability of the contract. Answer C is incorrect because the correspondence does not satisfy the Statute of Frauds. The elder cousin's March 1 letter was not an offer; she was merely wondering whether she should sell her house. The younger cousin's April 15 letter was in fact an offer, but the acceptance by the elder cousin was oral. Here, the party to be charged is the elder cousin and there is no writing signed by her. Answer D is incorrect. Whether the younger cousin was aware of the sale to the real estate broker does not affect the outcome of her suit. Had the elder cousin made an offer that had not yet been accepted, knowledge of the subsequent sale would work a revocation of the offer. However, it was the younger cousin who made the offer, which was accepted by the elder cousin by phone. There was no offer to be revoked on April 18.

On December 15, a law firm received from an office supply superstore an offer consisting of its catalog and a signed letter stating, "We will supply you with as many of the items in the enclosed catalog as you order during the next calendar year. We assure you that this offer and the prices in the catalog will remain firm throughout the coming year." On January 15, having at that time received no reply from the law firm, the superstore notified the law firm that effective February 1, it was increasing the prices of certain specified items in its catalog. Is the price increase effective with respect to catalog orders the superstore receives from the law firm during the month of February? A) No, because the superstore's original offer, including the price term, was irrevocable under the doctrine of promissory estoppel. B) No, because the original offer, including the price term, was irrevocable throughout the month of February. C) Yes, because the superstore received no consideration to support its assurance that it would not increase prices. D) Yes, because the period for which the superstore gave assurance that it would not raise prices was longer than three months.

B) No, because the original offer, including the price term, was irrevocable throughout the month of February. Answer B is correct. Office supplies are goods, and so UCC Article 2 applies to this contract. Under Article 2, if a merchant offers to buy or sell goods in a signed writing and the writing gives assurances that it will be held open, the offer is not revocable during the time stated (or a reasonable time if no time is stated) except in no event may the period exceed three months. Here, the superstore is a merchant who provided a signed written offer giving assurances that the offer would be held open for one year. If, as here, a merchant-offeror states that an offer will stay open for a period beyond the UCC's three-month limit on irrevocability, it will be bound only for three months. Thus, with respect to orders placed up until the end of March, the superstore is bound by the catalog prices. Answer A is incorrect. Promissory estoppel is an equitable doctrine that is applied, in the discretion of the court, upon a showing of foreseeable, substantial, and detrimental reliance. Nothing in the facts indicates that the law firm relied to its detriment on the superstore's promise. Answer C is incorrect. Unlike an option contract, no consideration is required to make a merchant's firm offer irrevocable for a period of up to three months. All that is required to hold open the offer is a written and signed promise by a merchant. Answer D is incorrect. The merchant's firm offer is irrevocable only for three months, but the fact the time promised exceeds three months does not make the firm offer void. Rather, at the end of the three-month period, the merchant is free to revoke the offer. February is within the three-month period of irrevocability; thus, the superstore may not change the prices during that month.

Selma sells printers. For all Jackson Parker Jet printers in her inventory, she paid her supplier $100 each. On August 1, by signed writing, Selma contracts to sell 100 Jackson Parker Jet printers to Bentell for $500 each, $50,000 in total. Hence, she expects to derive a profit of $40,000 [($50,000 - (100 x $100)]. Bentell agrees to take delivery and pay the purchase price on August 15. When August 15 arrives, Bentell tells Selma that he won't purchase the printers and thus commits an anticipatory repudiation. One hour later, Chase walks into Selma's showroom wanting to purchase 100 Jackson Parker Jet printers. Selma takes the printers designated for Bentell and sells them to Chase for $50,000. Selma then brings an action against Bentell seeking damages of $40,000—the profit she would have derived if Bentell had honored his contract. As for one more fact, please consider these alternatives scenarios: Before contracting with Bentell, Selma has exactly 100 Jackson Parker Jet printers and can acquire no more of them. Before and after contracting with Bentell, Selma has thousands of Jackson Parker Jet printers in her inventory, more than she will ever be able to sell. In which scenario should Selma recover the $40,000 she demands? Question 10 options: A) Scenario I, but not scenario II B) Scenario II, but not scenario I C) Both scenarios I and II D) Neither scenario I nor II

B) Scenario II, but not scenario I Answer B is correct here. If these were the last 100 printers and Selma ultimately had to sell them to Chase to mitigate damages after Bentell breached the contract, Selma would not be able to recover $40,000 in damages as she was able to mitigate this loss by selling the printers to Chase instead. No double-recovery will be permitted. If, instead, she has unlimited inventory of printers, which makes it hard to mitigate by selling to someone else, she has a claim for breach of contract against Bentell and should recover $40,000 in lost profits due to this breach.

A retailer of computers decided to purchase a certain model of high-speed printers from a wholesaler and then offer the printers for sale as part of a complete system with its keyboards and computers. Because printing speed was important, the written agreement between the wholesaler and the retailer provided that the printers must be able to print 40 pages per minute or more. The wholesaler was to provide 100 printers for the retailer at $300 each. The printers were to be delivered on March 15, and the retailer was to pay for the printers when they were delivered.When the shipment arrived, the retailer paid for it with a cashier's check for $30,000. However, when the retailer put one of the printers online with its system, it found that the printer was printing at a rate of only 30 pages per minute. The retailer then tested the entire shipment and found that none of the printers met the contract specifications. Although the printers did not meet the specifications, the retailer was under contract with a law firm to deliver 20 systems by April 1. The retailer negotiated with the law firm, and it was agreed that the retailer would install the 20 systems with the printers, but that the retailer would have to deliver by December 20 high-speed printers that met the contract specifications at no extra cost to the law firm, as replacements for these 20 printers. Immediately on making this agreement, the retailer sent an email to the wholesaler of the printers, telling the wholesaler that the shipment did not conform to the contract, but that the retailer was contractually obligated to use 20 of the printers.Because of the sale of the systems with the nonconforming printers to the law firm, how many printers has the retailer accepted? A) None of the printers, because the retailer has to replace 20 within six months. B) The 20 printers that the retailer sold to the law firm. C) The entire shipment of 100 printers. D) None of the printers, because the retailer seasonably notified the wholesaler of the nonconformity

B) The 20 printers that the retailer sold to the law firm. The retailer accepted the 20 printers that it sold to the law firm. A buyer's right to reject under the perfect tender doctrine generally is cut off by acceptance. If a buyer does any act inconsistent with the seller's ownership, it is considered an acceptance. The sale of the 20 nonconforming printers to the law firm, even for temporary use only, was thus an acceptance by the retailer. Note that the retailer is not without a remedy. Tender of nonconforming goods is a breach of contract, and even if the goods are accepted, the buyer is still entitled to damages resulting from the breach. (A) is incorrect because the fact that the 20 printers must be replaced does not affect the acceptance status of those printers. (C) is incorrect because if goods or their delivery fail to conform to the contract in any way, the buyer may reject all, accept all, or accept any commercial units and reject the rest. Thus, the fact that the retailer accepted 20 printers does not affect its ability to reject the other 80. (D) is incorrect because the seasonable notification, which is required for rejection (and thus required for the rejection of the 80 printers), does not affect the status of the 20 printers that were accepted.

A homeowner mailed a letter to a prospective buyer on January 15, offering to sell her house for $250,000. The letter was delivered to the buyer on January 17. The buyer mailed a letter to the homeowner on January 19, stating that she accepted the offer. The buyer's letter of January 19 operates as an acceptance even if which of the following is true? A) The homeowner telephoned the buyer on January 18, telling the buyer that she was revoking her offer. B) The buyer's properly posted letter was lost by the post office. C) The buyer misaddressed the envelope, and as a result, it was not delivered to the homeowner until February 1, after the homeowner had sold the property to a third party. D) The buyer knew that the homeowner had sold the property to a third party on January 18.

B) The buyer's properly posted letter was lost by the post office. Answer B is correct. Even if the post office loses the acceptance, a contract was formed. Under the mailbox rule, an acceptance becomes effective when a letter is properly posted. Any reasonable manner of acceptance is allowed unless the offeror provided otherwise. Here, there was no required means of acceptance, and the homeowner's mailing of her offer to the buyer would certainly authorize the buyer to use the same method in her acceptance. Answer A is incorrect. Unless an offer is irrevocable or a firm offer, an offeror retains the right to revoke the offer any time prior to acceptance by an offeree. Such revocation is effective upon receipt by the offeree. If the homeowner called the buyer on January 18, revoking the offer, the offer would be revoked and the attempted acceptance on January 19 would be too late. Under the mailbox rule, the buyer's acceptance would become effective when she mailed it on January 19, but that would be after the offer had been revoked on January 18. Answer C is incorrect. Under the mailbox rule, an acceptance becomes effective when the letter is properly posted. However, when, as here, the envelope is misaddressed, that rule no longer applies. In such a case, acceptance becomes effective only when received, which would be too late in this case. Answer D is incorrect. An offeree cannot accept an offer when she knows or has reliable information that an offeror has done some act, like selling the property, which is inconsistent with an outstanding offer. Revocation of an offer may occur prior to the time of acceptance, when the offeree acquires information that indicates to a reasonable person that the offeror no longer wishes to keep the offer open. Here, the buyer's knowledge that the homeowner sold her house to a third party, before the buyer accepted, would mean that the offer was revoked, and the buyer's power of acceptance would be terminated.

A large dairy regularly supplies milk to local grocery stores located in the same city. A large grocery store in the city, which had not previously done business with the dairy, sent a written order to the dairy for 50 cases of whole milk at the wholesale market price of $10 per case for prompt shipment. The warehouse clerk at the dairy negligently shipped 50 cases of low-fat milk instead.When the milk is delivered at the grocery store by a common carrier, which of the following is the most appropriate statement of the parties' rights and duties? A) The store must accept the delivery but is entitled to compensation for any damages caused by the nonconforming shipment. B) The dairy has breached the contract, and the store may refuse the shipment. C) Because the goods are nonconforming, the store can treat the delivery as a counteroffer and refuse acceptance. D) Because the goods are nonconforming, the store must reject the shipment and reship it to the dairy at the dairy's expense.

B) The dairy has breached the contract, and the store may refuse the shipment. Answer B is correct. The store's order is an offer. An offer to buy goods for current or prompt shipment may be accepted by either a promise to ship or by a shipment of conforming or nonconforming goods. The shipment of nonconforming goods is an acceptance creating a bilateral contract as well as a breach of the contract unless the seller seasonably notifies the buyer that a shipment of nonconforming goods is offered only as an accommodation. Here, a contract was formed and breached by the shipment. The dairy's act of shipping goods constituted an acceptance of the offer, but since the goods are nonconforming, the dairy breached the contract. Under the UCC perfect tender rule, the grocery store can accept all, reject all, or accept some and reject some. Thus, the store may refuse the entire shipment. Answer A is incorrect. A contract was formed and breached when the dairy shipped the wrong milk. The store need not accept the milk. When a seller delivers nonconforming goods, the buyer may accept all, reject all, or accept some and reject some. Answer C is incorrect. A shipment of nonconforming goods in response to an offer is a counteroffer only when it is accompanied by a notice that the seller is shipping the goods as an accommodation. In this case, the shipment constitutes an acceptance, rather than a counteroffer, because there was no notice that this nonconforming tender was shipped only as an accommodation. Answer D is incorrect. Under the perfect tender rule, when a seller ships nonconforming goods, the buyer may accept all, reject all, or accept some and reject some. The buyer is never required to reject the goods. Moreover, if the buyer rejects the goods, it is not required to reship the goods; it need only hold them for a reasonable time for the seller to remove them. Thus, the store has no responsibility to reship the milk to the dairy, even at the dairy's expense, if it decides not to accept delivery.

A park board in a large suburb announced that it was accepting bids for renovation work on its recreation center. A builder advertised for sub-bids for the electrical work, and a local electrician submitted to the builder by electronic bidding service a sub-bid of $130,000. However, due to the bidding service's negligence, the sub-bid that the builder received from the electrician read $30,000 instead of $130,000. Because this was the lowest sub-bid that the builder received for the electrical work, and $60,000 less than the next lowest sub-bid, the builder awarded the subcontract to the electrician. Based in part on the electrician's sub-bid, the builder came up with a bid for the job that beat out all of the competition and won the job.What is the electrician's best argument to successfully refuse to perform the resulting contract? A) The contract would be unconscionable. B) The great difference between the $30,000 figure and the next lowest bid should have alerted the builder to the existence of a mistake in the sub-bid. C) The electrician was not responsible for the negligence of the bidding service. D) The builder's own negligence in not checking out all sub-bids precludes enforcement of the contract.

B) The great difference between the $30,000 figure and the next lowest bid should have alerted the builder to the existence of a mistake in the sub-bid. The builder will not be permitted to snap up the electrician's offer because the great difference between the electrician's sub-bid, as transmitted, and the next lowest sub-bid should have alerted the builder to the obvious mistake in the electrician's sub-bid. Where only one of the parties entering into a contract is mistaken about facts relating to the agreement, the unilateral mistake will not prevent formation of a contract unless the nonmistaken party is or had reason to be aware of the mistake made by the other party, in which case the contract is voidable. Here, the difference between the $30,000 figure and the next lowest bid should have alerted the builder to the existence of a mistake, so the electrician should be able to refuse to perform the contract. (A) is incorrect because the concept of unconscionability concerns clauses in a contract that are so one-sided as to be unconscionable under the circumstances existing at the time the contract was formed. The concept is typically applied to one-sided bargains where one of the parties has substantially superior bargaining power and can dictate the terms of the contract to the other party. That is not what happened under the facts here. Rather, there was a unilateral mistake and, as discussed above, the builder should have known of the mistake. (C) is incorrect because, under the prevailing view, where there is a mistake in the transmission of an offer by an intermediary, the offer as transmitted is operative unless the other party knew or should have known of the mistake. Thus, the electrician would not be excused from performance based on the faulty transmission alone. (D) is incorrect because a builder has no duty to check out all bids and, thus, the builder's failure to check out the bids would not, by itself, be grounds for rescission.

A retailer telephoned a towel manufacturer and offered to buy for $5 each, a minimum of 500 and a maximum of 1,000, large bath towels to be delivered in 30 days. The manufacturer orally accepted this offer and promptly sent the following letter to the retailer, which the retailer received two days later: "This confirms our agreement today by telephone to sell you 500 large bath towels for 30-day delivery, /s/ The manufacturer." Twenty-eight days later, the manufacturer tendered to the retailer 1,000 (not 500) conforming bath towels, all of which the retailer rejected, because it had found a better price term from another supplier. Because of a glut in the towel market, the manufacturer cannot resell the towels except at a loss.In a suit by the manufacturer against the retailer, which of the following will be the probable decision? A) The manufacturer can enforce a contract for 1,000 towels, because the retailer ordered and the manufacturer tendered that quantity. B) The manufacturer can enforce a contract for 500 towels, because the manufacturer's letter of confirmation stated that quantity term. C) There is no enforceable agreement, because the retailer never signed a writing. D) There is no enforceable agreement, because the manufacturer's letter of confirmation did not state a price term.

B) The manufacturer can enforce a contract for 500 towels, because the manufacturer's letter of confirmation stated that quantity term. Answer B is correct. Since the confirmation letter stated 500 towels, this is the enforceable quantity term, and thus, the manufacturer can enforce a contract for 500 towels. A contract for the sale of goods for a price of $500 or more is within the Statute of Frauds and generally must be evidenced by a signed writing to be enforceable. In contracts between merchants, however, if one party, within a reasonable time after an oral agreement has been made, sends to the other party a written confirmation of the understanding that is sufficient under the Statute of Frauds to bind the sender, it will also bind the recipient if: (i) he has reason to know of the confirmation's contents; and (ii) he does not object to it in writing within 10 days of receipt. Here, the contract was for the sale of goods and was in excess of $500; therefore the Statute of Frauds must be satisfied. Since both the retailer and the manufacturer are merchants, a memorandum of the terms of the sale sent by the manufacturer to the retailer satisfies the Statute of Frauds unless the retailer objects within 10 days, which does not appear to be the case here. Answer A is incorrect. Because the confirmation stated 500 towels, this is the enforceable quantity term. As stated above, this contract is within the Statute of Frauds and must be evidenced by a signed writing to be enforceable. Here, because the parties are both merchants, the manufacturer's confirmation would suffice (see above). That confirmation listed the quantity as 500; therefore, that is the amount covered by the contract. Answer C is incorrect. The retailer is bound under the merchant's confirmatory memo rule. As explained above, in a contract between merchants, if one party, within a reasonable time after an oral agreement has been made, sends to the other party a written confirmation of the understanding that is sufficient under the Statute of Frauds to bind the sender, it will also bind the recipient if: he has reason to know of the confirmation's contents, and he does not object to it in writing within 10 days of receipt. Here, the parties are merchants and there is no indication that the retailer objected within 10 days. Thus, there was an enforceable contract. Answer D is incorrect. The only terms that need to be established to support a contract pursuant to the UCC are quantity and subject matter. UCC gap-filler provisions can establish those terms not specified by the parties (e.g., a reasonable price).

On November 5, an electronics store owner realized that his stock of 15 copies of the most popular video game of the holiday shopping season would not last until the first of the next month. Seeing an advertisement from the manufacturer of the game in a trade journal listing its price at $3,000 per hundred, with delivery one week from order, the store owner e-mailed to the manufacturer an order for 100 copies of the game at $3,000 per hundred. There were no further communications between the store owner and the manufacturer. By November 25, the store owner realized that the manufacturer was not going to deliver any of the video games. He thus was forced to obtain additional stock by purchasing from a middleman at a cost of $4,000 per hundred. The store owner brings an action for breach of contract against the manufacturer.Who will prevail? A) The manufacturer, because the communications between the parties were not definite or certain enough to form a contract. B) The manufacturer, because it never accepted the offer contained in the store owner's e-mail. C) The store owner, because his e-mail was an acceptance of the manufacturer's offer. D) The store owner, because he changed his position in reliance on the manufacturer's promise to deliver the video games within one week.

B) The manufacturer, because it never accepted the offer contained in the store owner's e-mail. The manufacturer will prevail because it never accepted the offer. For a communication to be an offer, it must contain a promise, undertaking, or commitment to enter into a contract, rather than a mere invitation to begin negotiations. The broader the communicating media, e.g., publications, the more likely it is that the courts will view the communication as merely the solicitation of an offer. An advertisement in a trade journal generally is construed as an invitation to submit offers, not an offer itself. It is an announcement of the price at which the seller is willing to receive offers. Thus, the store owner's e-mail was an offer that was never accepted by the manufacturer. (A) is wrong because the communications were definite and certain enough to form a contract. Under the UCC, only the quantity term must be definite and certain (or capable of being made so). Here, the quantity term, 100, was clear; the problem is that the offer containing it was not accepted. (C) is wrong because, as discussed above, the ad is not an offer. (D) is wrong because the ad in the trade journal was not a promise; hence, the store owner cannot rely on promissory estoppel or detrimental reliance to recover.

A dog owner owned a dog of little monetary value. One day, the dog disappeared, and the owner placed the following advertisement in the newspaper: "I will pay $500 to anyone who finds and returns my dog." A neighbor found the dog in his backyard late the next night. The man took the dog inside, intending to return it in the morning. Shortly before he returned the dog, the neighbor read the dog owner's advertisement in the morning newspaper. If the dog owner's advertisement was an offer, can the neighbor who found the dog recover $500 from the dog owner? A) Yes, because a contract was formed when he found the dog. B) Yes, because a contract was formed when he returned the dog to the dog owner. C) No, because he did not know of the offer when he found the dog. D) No, because of the Statute of Frauds.

B) Yes, because a contract was formed when he returned the dog to the dog owner. Answer B is correct. The neighbor can recover $500 because a contract was formed when he returned the dog. The neighbor has not accepted the offer until he completely performs the requested act, i.e., finding and returning the dog. Once he returns the dog, he has not only accepted the offer but fully completed his performance. At that point, the dog owner must perform his obligation under the contract and pay the neighbor. Answer A is incorrect. A contract is not formed until the neighbor fully completes his performance by returning the dog. Finding the dog alone does not entitle the neighbor to the reward money. The dog owner made a unilateral offer, i.e., one that requires acceptance by the doing of an act. Until the neighbor actually completes his performance by returning the dog, he is not entitled to the reward money. Answer C is incorrect. The neighbor's knowledge at the time of finding the dog is not material; only his knowledge at the time of performing the act required for acceptance (i.e., returning the dog) is relevant. Performance of an act requested by an offer, made before the offeree has knowledge of the offer, does not form a contract; however, when, as here, the offeree has only partially completed performance (finding the dog), completion of the requested performance (returning the dog) with knowledge acts as an acceptance. If the neighbor returned the dog without knowing of the dog owner's offer of a reward, he would have no contractual claim for the reward. Answer D is incorrect. This contract does not fall within the Statute of Frauds. The Statute of Frauds provides that certain contracts are enforceable only if evidenced by a written memorandum and signed by the party to be charged. The Statute of Frauds covers a contract for the sale of goods when the price is $500 or more, but this question does not involve a sale of goods—the neighbor who found the dog does not have title to the dog and is not selling the dog. Nor does this transaction come under any other provision of the Statute of Frauds. Therefore, no written, signed memorandum is necessary to enforce the dog finder's recovery.

In answer to a radio advertisement, a teenager two months shy of his 18th birthday contracted to buy a late model car from a car dealership. The agreement required a $1,500 down payment with the remainder of the $7,200 price to be paid in monthly installments to a local finance company. The teenager's first eight payments were made regularly until his driver's license was suspended. He then informed the company that no further payments would be forthcoming.The finance company sued for the remaining payments. The age of majority in the teenager's state is 18 years.Would the teenager be liable for the balance of the payments? A) Yes, because the car dealership was liable on the contract from the outset, notwithstanding his minority. B) Yes, because he kept the car for six months after reaching the age of majority. C) No, because he was a minor at the time of contracting, and the contract was voidable by him. D) No, because he informed the finance company in a timely manner after his driver's license was suspended.

B) Yes, because he kept the car for six months after reaching the age of majority. The court should rule that the teenager affirmed the contract and, thus, should be liable for the balance of the payments. Although infants generally lack capacity to enter into a contract that is binding on themselves, an infant may affirm, i.e., choose to be bound by his contract, upon reaching majority. Affirmance may be either express or by conduct, e.g., failing to disaffirm the contract within a reasonable time after reaching majority. Disaffirmance discharges all liability. Here, the teenager continued making payments on the car six months after reaching his 18th birthday, and did not disaffirm the contract. Thus, the court should hold that he affirmed the contract and was liable for the unpaid balance. (A) is incorrect because, although it is true that the car dealership was liable on the contract from the outset, that liability did not extend to the infant. A contract entered into between an infant and an adult is voidable by the infant but binding on the adult. (C) is incorrect because, although the teenager could have voided the contract while still a minor, he did not do so and, once he reached the age of 18, was left only with the option of disaffirming the contract within a reasonable time after reaching majority, which he also failed to do. Thus, his minor status at the time of contracting, without more, would not allow him to escape liability for the payments. (D) is incorrect because timely notice of a breach of contract does not mitigate the effects of his liability for the breach, and the teenager's affirmance of the contract, as discussed above, renders him liable for the unpaid balance.

The owner of a store in a small beach town contacted a manufacturer about buying 100 Adirondack chairs. The parties reduced their agreement to sell and buy the 100 chairs to a signed writing that contained all of the essential terms, as well as a merger clause. When the chairs were delivered to the store owner, she discovered that they were not "Adirondack chairs," as she understood that term and as that term was commonly used, but a different style of chair altogether that was manufactured in the Adirondack region of New York. The store owner refused delivery, and the manufacturer sued the store owner for breach of contract, arguing that he complied with the terms of the contract. Will the store owner be allowed to introduce evidence of the meaning of the term "Adirondack chair"? A) Yes, because the parol evidence rule does not apply to subsequent modifications. B) Yes, because of the dispute between the parties over the meaning of the term "Adirondack chair." C) No, because the written agreement of the parties was a complete integration. D) No, because the contract was for a sale of goods between merchants.

B) Yes, because of the dispute between the parties over the meaning of the term "Adirondack chair." The store owner will be allowed to introduce parol evidence of the meaning of the term "Adirondack chair" because there is a dispute as to its meaning. If there is uncertainty or ambiguity in the written agreement's terms or a dispute as to the meaning of those terms, parol evidence can be received to aid the factfinder in reaching a correct interpretation of the agreement. (A) is incorrect because a subsequent modification of the agreement is not in issue. There is a dispute as to the interpretation of a term in the original agreement. (C) is incorrect because, while it is probably true that the writing is a complete integration, that fact will not affect the admissibility of the evidence regarding the meaning of "Adirondack chair." Whether the writing was intended as a final expression of the contract or was a complete integration is important when parol evidence is sought to change the agreement, but the parties in this case are not trying to modify, add to, or subtract from the original written agreement. Instead, they are merely seeking clarification of a material term in the agreement. Parol evidence on this matter is admissible even if the writing is a complete integration. (D) is incorrect because the rule is not different for merchants with respect to interpreting a material term. In fact, Article 2 of the UCC is even more liberal in allowing, even in the absence of ambiguity, terms to be explained or supplemented by permitting evidence of course of dealing, usage of trade, and course of performance.

A ski-shop operator, in a telephone conversation with a glove manufacturer, ordered 12 pairs of ski gloves at the manufacturer's list price of $600 per dozen for delivery in 30 days. The manufacturer orally accepted the offer, and immediately e-mailed the operator this signed memo: "Confirming our agreement today for your purchase of one dozen pairs of ski gloves for $600, shipment in 30 days." Although the operator received and read the manufacturer's message within minutes after its dispatch, she changed her mind three weeks later about the purchase and rejected the conforming shipment when it timely arrived.On learning of the rejection, does the manufacturer have a cause of action against the operator for breach of contract? A) Yes, because the gloves were identified to the contract and tendered to the operator. B) Yes, because the manufacturer's memo to the operator was sufficient to make the agreement enforceable. C) No, because the agreed price was $600, and the operator never signed a writing evidencing a contract with the manufacturer. D) No, because the operator neither paid for nor accepted any of the goods tendered.

B) Yes, because the manufacturer's memo to the operator was sufficient to make the agreement enforceable. Answer B is correct. Because the contract was for goods worth $500 or more, the Statute of Frauds must be satisfied. In contracts between merchants, if one party, within a reasonable time after an oral agreement has been made, sends to the other party a written confirmation of the understanding that is sufficient under the Statute of Frauds to bind the sender, it will also bind the recipient if: (i) he has reason to know of the confirmation's contents; and (ii) he does not object to it in writing within 10 days of receipt. Here, both the operator and the manufacturer are merchants, and the memo containing the terms of the sale sent by the manufacturer to the operator satisfies the Statute of Frauds to bind the manufacturer. There is no indication that the operator objected within 10 days, so the agreement is enforceable against the operator. Answer A is incorrect. Because the contract was for goods worth $500 or more, the Statute of Frauds must be satisfied. Merely identifying the goods to the contract and tendering the goods does not satisfy the Statute of Frauds. Answer C is incorrect. As stated in the explanation to answer B above, the memo made the contract enforceable. Under the merchant's confirmatory memo rule, the agreement is enforceable against the operator even though she never signed it. Answer D is incorrect. This choice is incorrect because, as stated above, the manufacturer can recover because of the merchant's confirmatory memo rule. Neither payment for nor acceptance of goods is necessary to make a contract enforceable. This choice alludes to an exception to the Statute of Frauds. When goods are accepted or paid for; even if there is no writing, the contract is enforceable for the quantity of goods accepted or paid for. However, this exception is irrelevant here because the goods were not accepted or paid for and so this exception does not apply.

The owner of an athletic shoe company negotiated a contract with the head of an advertising agency, under which the advertising agency agreed to handle the shoe company's account for a six-month period. The parties also agreed that the shoe company's marketing director would prepare a written version of their agreement, which each party would sign. The marketing director prepared the document, and the owner of the shoe company sent it to the advertising agency, without signing it. The head of the advertising agency read the document, signed it, and deposited it in the mail. Before the written agreement reached the shoe company's owner, he changed his mind and called the advertising agency to cancel the deal.If the advertising agency sues the shoe company for damages resulting from breach of contract, will it prevail? A) Yes, because an enforceable contract was formed when the document was deposited in the mail by the head of the advertising agency. B) Yes, because the two parties entered into a valid, enforceable contract on behalf of their two organizations. C) No, because there was no valid acceptance of an offer prior to the shoe company's rejection. D) No, because there is no written memorandum signed by the party to be charged.

B) Yes, because the two parties entered into a valid, enforceable contract on behalf of their two organizations. Answer B is correct. In this case, the facts indicate that the oral negotiations between the two parties resulted in an enforceable oral contract. The writing was merely an optional memorialization of their agreement. This contract could be performed within a year (it was a six-month contract), and so it did not have to be in writing to be enforceable under the Statute of Frauds. Answer A is incorrect. This choice refers to the mailbox rule under which an offer is accepted when it is properly mailed. However, here, the deposit of the agreement in the mail was not an acceptance by the head of the advertising agency, because the contract had already been formed. Answer C is incorrect. A valid contract was formed when the parties orally agreed to the arrangement. This agreement was before the attempted repudiation by the owner of the shoe company. Answer D is incorrect. This contract is not subject to the Statute of Frauds. It can be performed within one year from its making and does not involve suretyship, land, or the sale of goods in excess of $500. In this case, the oral negotiations between the parties resulted in an enforceable oral contract. The writing was merely an optional memorialization of their agreement.

By properly structured signed writing, Xavier and Yelena form a contract under which Xavier is to sell his realty to Yelena for $500,000. When the time comes to close the transaction, Yelena tenders the purchase price, but Xavier refuses to hand her his deed, insisting that he now wants more than $500,000 for the land. Xavier thus breaches his contract. Pelco and Wright form a contract under which Pelco will publish Wright's novel. Among other of its many provisions, the contract requires that "Pelco register the relevant copyright jointly in both parties' names, each to be equal owner with the other." Pelco honors all of the contract's provisions except that one. He registers the copyright in his own name only. I. Yelena is probably entitled to a decree of specific performance ordering Xavier to convey his realty to Yelena (on condition that she pay the $500,000 purchase price). II. Wright is probably entitled to a decree of specific performance compelling Pelco to amend the copyright registration so to show Wright as a joint, equal owner. Which statement is correct? Question 4 options: A) I but not II B) II but not I C) Both I and II D) Neither I nor II

C) Both I and II Both contracts are enforceable and have been breached. Specific performance is appropriate in both. Generally, specific performance is appropriate if the legal remedy (damages) is inadequate. The legal remedy (damages) generally is inadequate when the subject matter of the contract is rare or unique. The rationale is that if the subject matter is rare or unique, damages will not put the nonbreaching party in as good a position as performance would have, because even with the damages the nonbreaching party would not be able to purchase substitute performance. Specific performance is always available for land sale contracts because all land is considered to be unique. So, here, Yelena is probably entitled to specific performance in the contract involving purchase of land. Also, Wright is probably entitled to decree of specific performance compelling Pelco to amend the copyright registration (note, this is not a personal services contract; instead, this is a contract to publish a novel, and it would not be difficult for the court to supervise whether Pelco has complied with the order of the court to amend the copyright registration).

A breeder bought a two-month-old registered boar at auction from a livestock supplier for $800. No express warranty was made. Fifteen months later, tests by experts proved conclusively that the boar had been born incurably sterile. If this had been known at the time of the sale, the boar would have been worth no more than $100.In an action by the breeder against the livestock supplier to avoid the contract and recover the price paid, the parties stipulate that, as both were and had been aware, the minimum age at which the fertility of a boar can be determined is about 12 months.Is the breeder likely to prevail? A) Yes, because the parties were mutually mistaken as to the boar's fertility when they made the agreement. B) Yes, because the livestock supplier impliedly warranted that the boar was fit for breeding. C) No, because the breeder assumed the risk of the boar's sterility. D) No, because any mistake involved was unilateral, not mutual

C) No, because the breeder assumed the risk of the boar's sterility. Answer C is correct. When both parties entering into a contract are mistaken about existing facts relating to the agreement, the contract may be voidable by the adversely affected party if: the mistake concerns a basic assumption on which the contract is made, the mistake has a material effect on the agreed-upon exchange, and the party seeking avoidance did not assume the risk of the mistake. If the parties knew that their assumption was doubtful (i.e., they were consciously aware of their ignorance), the adversely affected party will be deemed to have assumed the risk. To be a defense to the contract, it must be a mistake, not a mere uncertainty. Here, the parties stipulated that they knew that there was no way of knowing whether a boar of this age was fertile. Thus, the breeder assumed the risk that the boar was infertile, and cannot avoid the contract. Answer A is incorrect because even if the parties were mistaken as to the boar's fertility, the breeder assumed the risk of that mistake. Here, the parties stipulated that they were both aware that the boar's fertility was impossible to know at two months. Thus, the breeder assumed the risk that the boar was infertile Answer B is incorrect because even if the breeder were able to establish a breach of warranty, the remedy would be damages (the difference between the value of the goods as delivered and the value of the goods as warranted), not avoidance of the contract and return of purchase price. Answer D is incorrect. The mistake here was mutual; neither party could know the fertility of the boar. Also, the breeder would be unable to avoid the contract under either theory of mistake because he assumed the risk when he bought the boar knowing that there was no way of telling whether it was infertile.

An air conditioner distributor had three air conditioners that he wanted to sell to clear his storage facilities. On July 1, he sent a signed memo to a local businessman, saying "I have three air conditioners remaining. I will sell them to you for $1,000 each. I will hold this offer open until July 31." On July 27, another regular customer of the distributor asked if he had any air conditioners remaining for sale. The regular customer offered to purchase two of the remaining three air conditioners, and the distributor sold them, leaving him with only one air conditioner. The distributor then informed the local businessman of the sale and asked if he wanted the last air conditioner. The local businessman immediately sent an e-mail to the distributor demanding that all three air conditioners be sent. The distributor responded that he was unable to do this. The local businessman brought suit against the distributor for breach of contract. If the local businessman prevails, it will most likely be because which of the following? A) The distributor did not communicate his revocation of the offer to the local businessman before selling the air conditioners to his regular customer. B) A contract was formed between the distributor and the local businessman, which was breached by the distributor's sale of the air conditioners to his regular customer. C) An option contract was formed, which gave the local businessman the exclusive option of purchasing the air conditioners until July 31. D) The distributor's memo constituted a merchant's firm offer, which was not revocable.

D) The distributor's memo constituted a merchant's firm offer, which was not revocable. Answer D is correct. The distributor is a merchant. He executed a written and signed offer for the sale of goods and promised to keep the offer open for one month. Under UCC §2-205, this offer is a firm offer, which is irrevocable for the time stated (not in excess of three months), even though it is not supported by consideration. Answer A is incorrect. While it is true that the distributor did not communicate his revocation of the offer, this choice is incorrect because the distributor had made a merchant's firm offer and therefore could not revoke his offer until after July 31. (See choice D explanation above.) Answer B is incorrect. The local businessman had not accepted the distributor's offer before July 27, and therefore no binding contract had been formed. The contract formation did not occur until the local businessman actually manifested his assent to the distributor's firm offer. The facts do not indicate any such acceptance by the businessman prior to July 27; therefore, no contract had been formed at the time the distributor sold the air conditioners to the customer. Answer C is incorrect. An option requires consideration, and the local businessman gave none. When an offeror promises to keep an offer open for a specified period of time, and the offeree gives consideration for this promise, the offer is irrevocable during this time, and an option contract arises. The person furnishing the consideration must do something or promise to do something that he does not legally have to do, or he must promise to forbear or actually forbear from doing something he has the legal right to do. Here, there is no option contract, because the local businessman has not furnished any consideration. Although a merchant's firm offer is similar to an option contract, it is not an option contract.

A retailer entered into an oral contract with an office supply wholesaler to buy 100 file boxes for an upcoming back to school sale at the retailer's store. The wholesaler agreed to deliver the file boxes in two weeks at a cost of $4 per file box. A week later, the retailer phoned the wholesaler and asked if she could increase her order to 200 file boxes. The wholesaler agreed. The wholesaler delivered the 200 file boxes as promised, but the retailer accepted only 150 upon discovering that she lacked storage space for all 200.May the wholesaler recover damages with respect to the 50 file boxes that were not accepted? A) Yes, because the retailer accepted $600 worth of file boxes. B) Yes, because the modification was for less than $500. C) No, because the contract as modified was for $800. D) No, because the wholesaler is a merchant with respect to file boxes.

C) No, because the contract as modified was for $800. The wholesaler may not recover damages. Under the UCC Statute of Frauds, a contract for the sale of goods for $500 or more is unenforceable unless evidenced by a writing signed by the party sought to be held liable. The original contract was for $400 and, thus, was not within the Statute. Whether a modification must be in writing to be enforceable depends on whether the entire contract price as modified is within the statute. Here, the retailer and the wholesaler modified their original contract to 200 file boxes, bringing the total price to $800. Thus, the modification was unenforceable under the Statute and Frauds, and the wholesaler cannot collect damages with respect to the 50 unaccepted file boxes. (A) is incorrect. Acceptance is an exception to the Statute of Frauds—but only to the extent of the goods accepted. That is, an oral contract for the sale of goods for $500 or more is enforceable to the extent the goods are accepted. Here, the original contract for 100 boxes is enforceable, but the modification is not. However, since the retailer accepted 50 additional boxes, the modified contract is enforceable to the extent of the additional 50 boxes accepted. The fact that the accepted amount meets the Statute of Frauds $500 threshold does not make the contract enforceable for all 200. (B) is incorrect because, as noted above, when determining whether a contract for the sale of goods is enforceable, we look at the whole contract price as modified; the price of the modification itself does not matter. (D) is incorrect because the fact that the wholesaler is a merchant with respect to the goods being sold (file boxes) has no bearing on the enforceability of the contract here.

An art gallery owner prominently displayed a painting of a classic car by an artist who specializes in American pop culture. A collector, who used to own a similar car, visited the gallery on February 1, admired the painting, and told the gallery owner that she would like to own it one day. On February 2, the gallery owner wrote to the collector, "I know you liked the painting you saw yesterday. I would be willing to let it go for $2,500." The letter was mailed that day and received by the collector on February 4. On February 3, the collector wrote to the gallery owner, "I offer to buy the painting I saw in your gallery for $2,500." The letter was mailed on February 3 and received by the gallery owner on February 5. On February 6, a tourist visited the gallery and offered $3,000 for the painting. The gallery owner accepted. The collector visited the gallery on February 7 with $2,500, intending to pay for the painting. As soon as she greeted the gallery owner, the gallery owner informed her that he had already sold the painting. If the collector sues the gallery owner for breach of contract, will she prevail? A) Yes, because her letter accepted the gallery owner's offer. B) Yes, because the gallery owner made a firm offer that could not be revoked before the collector's February 7 visit to the gallery. C) No, because the gallery owner revoked his offer before the collector accepted it. D) No, because the gallery owner's letter was too vague to be an offer.

C) No, because the gallery owner revoked his offer before the collector accepted it. Answer C is correct. A contract requires mutual assent, which is an agreement on the "same bargain at the same time"— "a meeting of the minds." Two offers of the same thing are not the same as an offer and acceptance—there is no intent to conclude the bargain. To accept an offer, a party must be aware of the offer. Here, the letters of the gallery owner and the collector containing offers crossed in the mail. Therefore, neither letter operated as an acceptance of the other's offer. Because the gallery owner did not agree to keep her offer open for any period of time, it was revocable at will and it was revoked before the collector accepted it. Therefore, there is no contract, and the gallery owner will prevail. Answer A is incorrect. The letters of the collector and the gallery owner crossed in the mail. The collector did not know of the gallery owner's offer at the time she made an identical offer to the gallery owner (and vice versa). Therefore, the collector's letter does not operate as an acceptance. Likewise, the gallery owner's letter does not constitute an acceptance of the collector's offer. The gallery owner effectively revoked her offer before the collector accepted, so the collector cannot sue on a contract. Answer B is incorrect. The gallery owner is a merchant who made a written offer, but it was not a merchant's firm offer because she did not promise to keep it open for any period of time. Her offer could be revoked at will and it was revoked before the collector accepted it. Therefore, there is no contract between the collector and the gallery owner. Answer D is incorrect. This answer choice reaches the right result for the wrong reason. While the gallery owner's letter is not specific about the exact painting, it does refer to an objective event, which identified the painting that the gallery owner wanted to sell and is therefore specific enough to be a legal offer. That offer, however, was revoked before it was accepted, and therefore no contract resulted.

A general contractor was preparing to bid on a contract to construct a state office building. She solicited bids on the masonry work from two masonry subcontractors. The first masonry firm submitted a bid of $75,000. The second masonry firm's bid was $80,000. The general contractor used a figure of $75,000 for the masonry work in preparing her successful bid to construct the office building. However, because the second masonry firm has worked with the general contractor on past jobs, the general contractor desired to hire it as the masonry subcontractor. The general contractor offered the job to the second masonry firm for $72,500, and it accepted. If the first masonry firm sues the general contractor to collect the profit that it would have made on the job, will it prevail? A) Yes, because its irrevocable bid ripened into a contract when the general contractor won the contract. B) Yes, because, as the low bidder, it entered into a binding contract with the general contractor, subject to the condition precedent that the general contractor win the bid. C) No, because the general contractor never accepted the first masonry firm's offer. D) No, because the second masonry firm agreed to perform the masonry work at a price that was lower than the first masonry firm's bid.

C) No, because the general contractor never accepted the first masonry firm's offer. Answer C is correct. A subcontractor bid is only an offer, which need not be accepted by a general contractor, even though the subcontractor is obligated to keep the offer open. The custom in the construction industry is that a subcontractor, making a bid on which a general contractor relies in making a general bid, implicitly agrees to keep that offer open for a reasonable time after the contract is awarded. Because a general contractor relies to its detriment on a subcontractor's bid in making a general bid, promissory estoppel enforces the subcontractor's promise to keep the offer open. In effect, promissory estoppel turns the subcontractor's bid into an option contract, which the general contractor may in turn make into a binding contract by accepting the subcontractor's bid within a reasonable time after the general contract is awarded. Until the general contractor actually accepts the subcontractor's bid, however, no binding contract for performance of the work is formed. Answer A is incorrect. As explained above, the first masonry firm's bid was irrevocable for a reasonable time; however, it never ripened into a contract to perform the masonry work because its bid (offer) was not accepted. Answer B is incorrect. Although the first masonry firm was the low bidder, and the general contractor used its bid to win the general contract, this alone does not mean that a binding contract for the performance of the masonry work was formed between the general contractor and the first masonry firm or that such a contract was formed subject to a condition precedent. Answer D is incorrect. The general contractor will prevail, but not because the second masonry firm agreed to do the work for a lower price than the first masonry firm's bid. The general contractor will prevail, because she never accepted the first masonry firm's offer. The important issue here is that the general contractor never accepted the first masonry firm's bid, not whom she hired to do the work once she won the general contract. The motive for the rejection of the bid is irrelevant, because the general contractor was not bound to the lowest bidder. In fact, the general contractor could contract with the second masonry firm to perform the masonry work at a higher price than that contained in the first masonry firm's bid.

A general contractor who wished to bid on a construction project solicited bids from a variety of subcontractors. Four electrical subcontractors submitted bids to the contractor in the amounts of $75,000, $85,000, $90,000, and $95,000, respectively.As he was making out his company's bid, which was higher than he wanted it to be, the contractor called the low bidder on the electrical work and told him, "We won't be able to do it with your present bid, but if you can shave off $5,000, I'm sure that the numbers will be there for us to get that project." The low bidder told the contractor that he could not lower his bid, adding that the bid he submitted was based on a $15,000 error, and he could not do the job for less than $90,000. The contractor lost the construction job and subsequently sued the low bidder.For what is the low bidder is liable? A) Breach of contract, because the mistake was not so unreasonably obvious as to make acceptance of his bid unconscionable. B) Breach of contract, because the mistake was unilateral. C) Nothing, because the low bidder rejected the contractor's counteroffer. D) Nothing, because even though the low bidder lacked authority to renege on its bid, the contractor suffered no damages because no bidder was willing to do the work for $70,000.

C) Nothing, because the low bidder rejected the contractor's counteroffer. The low bidder is liable for nothing because no contract was formed between the contractor and the low bidder. Formation of a contract requires mutual agreement between the parties (offer and acceptance) and consideration. There was no contract here because there was no acceptance. The low bidder's bid constituted an offer—a certain and definite promise, undertaking, or commitment to enter into a contract communicated to the offeree. An offer gives the offeree the power to accept and create a contract until the offer is terminated. An offer can be terminated in a number of ways, including through a counteroffer from the offeree. A counteroffer serves as both a rejection terminating the original offer and a new offer from the original offeree, thus reversing the former roles of the parties and giving the original offeror the right to accept or reject the new offer. Here, the contractor's call constituted a rejection and a counteroffer that the low bidder rejected, and so no contract was formed. Therefore, the low bidder cannot be held liable. (A) and (B) are incorrect because, as stated above, a contract was never formed between the contractor and the low bidder. Thus, it is irrelevant whether the mistake was unilateral or obvious. (Note that the general rule is that a contract will not be set aside for a unilateral mistake unless the nonmistaken party either knew or should have known of the mistake. Thus if the contractor had not called the low bidder but had instead accepted his bid, the low bidder would be liable on the contract despite his mistake because (B) the mistake was unilateral and (A) it was not obvious.) (D) is incorrect for several reasons: First, it relies on the existence of a contract, and as stated above, there is no contract here. Second, the premise that the low bidder could not renege on its offer is untrue. The general rule is that offers are revocable until accepted. In a subcontractor bid situation, a bid is treated as irrevocable for a reasonable amount of time because of detrimental reliance (i.e., the general contractor will rely on the mistaken bid in preparing his bid). However, here the contractor learned of the low bidder's mistake before any reliance on the bid. Moreover, it is unclear whether the contractor is complaining about the low bidder's reneging on his bid; the contractor appears to be complaining that the low bidder refused to lower the mistaken bid. The low bidder would have no duty to lower its bid in any case. The final premise in (D) is irrelevant. If the low bidder lacked the power to renege, the lack of power goes to the $75,000 bid; the fact that no one would do the job for $70,000 has no bearing on the issue.

At the start of this past baseball season, a wholesale baseball distributor and a baseball team entered into a written agreement, whereby the distributor agreed to supply baseballs, meeting league specifications, to the team at a price of $20 per dozen, in such quantities as the team would order from time-to-time. The distributor agreed to deliver the balls to the team's ballpark within 10 days of each order, payment to be cash on delivery. At first, the team used the balls left over from the previous season and ordered nothing. Then on April 19, the team placed a written order with the distributor for 50 dozen baseballs, at $20 per dozen, to be delivered on May 1. Since the team knew it would need an additional 50 dozen balls by the last two weeks in May, the team enclosed with the order a letter stating, "We also order an additional 50 dozen balls, if we can buy them for $18 per dozen."What is the state of the contractual relationship between the parties at the time the baseball distributor received the April 19 communication from the team? A) No contractual relationship exists, because the original agreement is unenforceable for lack of consideration. B) There is an enforceable contract for only 50 dozen baseballs at $20 per dozen; the language regarding the additional 50 dozen baseballs is stricken. C) There is an enforceable contract for the entire May requirements of the team, namely 100 dozen baseballs, at $20 per dozen. D) There is an enforceable contract for 50 dozen baseballs at $20 per dozen, and an offer from the team to purchase an additional 50 dozen baseballs at $18 per dozen.

D) There is an enforceable contract for 50 dozen baseballs at $20 per dozen, and an offer from the team to purchase an additional 50 dozen baseballs at $18 per dozen. Answer D is correct. The original agreement between the team and the distributor does not constitute a contract because there was no consideration on the part of the team; the team was under no obligan to order any baseballs at all. (This is not a requirements contract, because the team has not promised to purchase all of their baseballs from the distributor.) The agreement was actually only an offer by the distributor to sell baseballs for $20 per dozen. Until the distributor revokes this offer, it may be accepted. When the team ordered 50 dozen balls on April 19, it accepted the offer and a bilateral contract was formed for the purchase and sale of that 50 dozen balls. The remainder of the communication from the team to the distributor is an offer to purchase additional balls at $18 per dozen. The distributor may choose to accept or reject the team's offer. Answer A is incorrect. Although the original agreement was unenforceable, a contract was formed when the team accepted the distributor's offer, at least as to the quantity ordered under the terms offered. The distributor offered to sell the team baseballs at $20 per dozen; once the team accepted that offer by ordering 50 dozen balls on April 19, the parties became bound by a contract for the sale of that 50 dozen balls. In effect, the original written agreement between the parties was only an offer from the distributor. The team's signature was of no effect. However, by ordering some baseballs on the terms offered by the distributor, the team accepted the offer, there was consideration on both sides, and a bilateral contract was formed. Answer B is incorrect. The language concerning the additional 50 dozen baseballs is not stricken. That language constitutes an offer to buy 50 dozen baseballs at $18 per dozen. The distributor may or may not accept that offer. Answer C is incorrect. This is not a requirements contract obligating the team to purchase all of their requirements from this distributor at $20 per dozen. Under a requirements contract, a buyer agrees to buy and the seller agrees to sell all of a particular item that the buyer requires. Thus, the buyer is agreeing not to purchase the item from any other seller. Here, the original agreement between the parties does not obligate the team to purchase any baseballs at all from the distributor, let alone all of its baseballs. In effect, this is only an offer from the distributor, which the team signed for no legal reason. If the distributor does not accept the team's offer to purchase the additional 50 dozen balls at $18 per dozen, then a contract has been formed only as to the original 50 dozen balls ordered on April 19, pursuant to the distributor's original offer to sell balls to the team at $20 per dozen.

An automobile manufacturer entered into a contract with a well-known tire distributor. The contract provided that the distributor would deliver to the automobile manufacturer 1,000 tires on the 15th of each month for $50,000 per shipment, with payment due upon receipt.The automobile manufacturer and the tire distributor properly fulfilled their contractual obligations for two months. The day after the third tire delivery, the tire distributor's president visited the automobile manufacturer and found the automobiles produced with his company's tires to be a "disgrace." To protect the tire distributor's reputation, the president announced that he will not send any additional tire shipments. The automobile manufacturer immediately brought a breach of contract action against the tire distributor.Which party is likely to prevail in the breach of contract suit? A) The tire distributor, because it was not in breach at the time the action was filed. B) The tire distributor, if it can show that its reputation would be harmed by having its tires associated with the automobiles. C) The automobile manufacturer, because the president's action is an anticipatory repudiation. D) The automobile manufacturer, because the president's action amounts to a prospective inability to perform.

C) The automobile manufacturer, because the president's action is an anticipatory repudiation. The refusal of the tire distributor's president to send any additional tire shipments to the automobile manufacturer amounted to an anticipatory repudiation, which breaches the contract. An anticipatory repudiation arises where a promisor, by words or conduct, unequivocally indicates that he cannot or will not perform when the time comes. In such a case, the other party has several options, including treating the repudiation as an immediate breach and suing for damages. Thus, (C) is correct and (A) is incorrect. (B) is incorrect because the "disgraceful" quality of the automobiles and the possible detriment to the tire distributor's reputation would not be a defense under the contract because the contract made no provision regarding the quality of the automobile manufacturer's product. (D) is incorrect because the president's actions were an anticipatory repudiation and not a prospective inability to perform, which involves conduct or words that merely raise doubts that the party will perform and do not rise to the level of an unequivocal refusal to perform. Here, the president refused without equivocation to send any additional tire shipments to the automobile manufacturer. A prospective inability to perform would not permit the automobile manufacturer to treat it as an immediate breach. It would give it the right to demand assurances.

A young man proposed to his girlfriend, but she was reluctant because of his meager income and lack of job potential. The young man told his father about her reluctance. The father told the girlfriend that if she married his son, he would support them for six months and send his son to a six-month computer technology training school. This was sufficient to dispel her reservations and the two were married very soon after. When they returned from their honeymoon, the father refused to go through with his offer. Although the girlfriend is happy in her marriage, she sued the father for damages.If the father prevails, what is the likely reason? A) The father's promise was not supported by valid consideration. B) The contract is against public policy. C) The contract was oral. D) The girlfriend is happy and therefore has incurred no detriment.

C) The contract was oral. If the father prevails, it will be because his promise to the young woman was not in writing. The Statute of Frauds requires that a contract in consideration of marriage must be evidenced by a writing to be enforceable. This includes any promise that induces someone to marry by offering something of value. Hence, the father's offer of support and education expenses is unenforceable. (A) is wrong because a promise to marry is a sufficient detriment to constitute valid consideration. (B) is wrong because there is no public policy against encouraging marriage. (D) is wrong because a person's pleasure or displeasure from performing a contractual duty is irrelevant as to whether performance constitutes consideration. The young woman agreed to do something that she was under no legal obligation to do in exchange for the father's promise of financial support; hence, consideration exists.

A department store buyer and a manufacturer of food processors entered into a written contract whereby the manufacturer would sell to the buyer 50 of its top-of-the-line models for $100 each. When the delivery arrived on May 15, several days early, the buyer noticed that the food processors were a different model that did not have all of the features as the top-of-the-line model that was ordered. The buyer contacted the manufacturer and told him that he was rejecting the food processors that were delivered to him and expected the manufacturer to send 50 top-of-the-line models immediately. The manufacturer replied that because of a backlog of orders that had not yet been filled, the top-of-the-line models could not be delivered until August 15. Because the department store had contracted with a restaurant to deliver three top-of-the-line models by May 31, the buyer delivered three of the nonconforming food processors along with a promise to replace them with three top-of-the-line models in mid-August. The buyer returned the remaining food processors to the manufacturer. How much could the department store recover from the manufacturer for the three food processors that it delivered to the restaurant? A) Nothing, because they were resold to another. B) Nothing, because it accepted them knowing they were defective. C) The difference between the market price of the top-of-the-line models and the existing food processors' actual value. D) The difference between the existing food processors' actual value and the cost of the food processors that the department store must provide to the restaurant in mid-August.

C) The difference between the market price of the top-of-the-line models and the existing food processors' actual value. The department store was entitled to recover contract damages from the manufacturer for the three food processors that it accepted. If the buyer accepts goods that breach one of the seller's warranties, the basic measure of damages is the difference between the value of the goods as delivered and the value they would have had if they had been according to the contract, which is best stated by choice (C). The department store's acceptance of the three food processors did not waive its right to collect damages for the defect in quality. Thus, (A) and (B) are wrong. Having accepted the nonconforming food processors, the department store's damages would be the difference between the value of the food processors as received and what they would have been worth if they had been as warranted, plus foreseeable incidental and consequential damages. (D) is wrong because the measure of damages is based on market value rather than cost. Also, the agreement with the restaurant was to accommodate the department store only and was not foreseeable by the manufacturer.

A contract between a camera store and a distributor of camera lenses called for the sale of 20 standard zoom lenses that would be compatible with a particular manufacturer's film and digital cameras, for a price of $300 per lens. After an employee of the camera store with authority to accept deliveries accepted shipment of the zoom lenses, store personnel learned that the lenses would not work with the manufacturer's film camera, but instead were compatible only with the manufacturer's digital camera. The camera store filed suit. At trial, the store's attorneys established that the nonconforming lenses were worth $200 per lens. Which of the following should be the measure of damages applied to determine the camera store's loss? A) The cost of purchasing lenses compatible with the manufacturer's film camera from another supplier. B) The contract price of $6,000, because the camera store has not received the benefit of its bargain. C) The difference between the value of the lenses as received and their value if they had been compatible with the manufacturer's film camera—$2,000. D) The camera store is not entitled to damages, because its employee accepted the lenses as shipped by the distributor.

C) The difference between the value of the lenses as received and their value if they had been compatible with the manufacturer's film camera—$2,000. The camera store's measure of loss would be the difference between the value of the goods as received and the value they would have had if they had conformed to the contract. When goods are delivered that do not conform to the parties' contract, the buyer has the option to accept all, reject all, or accept any commercial units and reject the rest. If the buyer chooses to accept the goods, as the camera store did here, the buyer has a right to recover damages for the nonconformity. The standard measure of damages as to accepted goods is the difference between the value of the goods as delivered and the value they would have had if they had been conforming (plus incidental and consequential damages). Here, the goods as delivered were worth $4,000 ($200 per lens for 20 lenses) and they would have been worth $6,000 ($300 per lens for 20 lenses) if they had been as the contract required. Thus, (C) is correct. (A) is incorrect because it describes the remedy of "cover," and cover is available only when the buyer rejects the goods. (If the buyer rejects the goods, he may either cancel the contract or buy substitute goods and charge the breaching seller for the substitutes.) (B) is incorrect because the buyer is entitled to return of the entire contract price only when he cancels the contract. Otherwise, the buyer would get a windfall based on the value of the goods received. (D) is incorrect because, as indicated above, a buyer who accepts nonconforming goods is still entitled to damages.

An actor was hired by a director to perform in a three-month run of a play that the director had written. Two weeks after the play's opening night, the actor fainted and could no longer perform because of a resulting concussion. After a week of treatment, the actor returned to the theatre to resume his role but the director refused him admission, having hired someone else to replace the actor.In his action by the actor against the director for breach of contract, which of the following, if proved, would be the director's best defense? A) Most of the theatre members felt that the replacement was a better performer than the actor. B) The director had offered the actor a job in the ticket office at a higher salary, but the actor had declined. C) The director could not find any substitute except for the replacement actor, who demanded a contract for a minimum of two and a half months if he was to perform at all. D) The actor did not really wish to continue to be employed by the director, but is merely seeking money.

C) The director could not find any substitute except for the replacement actor, who demanded a contract for a minimum of two and a half months if he was to perform at all. The director is entitled to find a substitute to perform in the actor's absence. If the only way the director could acquire a substitute was to agree to the extended term, then the director's actions would be proper. (A) is wrong because this would not relieve the director from liability to the actor. (B) is wrong because the actor does not have to accept any job under the contract, only the job that was the subject of the contract or a similar job to avoid damages. (D) is wrong because even if the actor did not wish to continue in the role, this factor would not excuse the director from his liability under the contract.

A wholesale seller of turquoise received a fax from a retail seller of Indian jewelry, a long-time customer, which read: "Send 200 16-inch strands of Kingman turquoise beads at your usual price of $45 per strand." The wholesaler faxed the customer back a confirmation, promising to ship the strands within 10 days. However, the wholesaler currently had no strands in stock and so began making phone calls to locate them in sufficient quantity. Ultimately, he secured the strands of beads. However, before he could ship the beads to the retailer, he received another fax from the retailer canceling the order without explanation. The wholesaler immediately set out to find another buyer for the beads, but found that Kingman turquoise had fallen out of favor. Consequently, the wholesaler was forced to sell the strands of beads at a "salvage" price of $4,000. If the wholesaler sues the retailer for damages, how much should he receive? A) Nothing, because this was a contract between merchants and the retailer canceled within a reasonable time. B) The full contract price of $9,000 (because the retailer breached the contract and the price was fair), plus the wholesaler's incidental expenses. C) The full contract price plus incidentals, less the $4,000 salvage price. D) The full contract price, less the $4,000 salvage price.

C) The full contract price plus incidentals, less the $4,000 salvage price. The wholesaler is entitled to recover the incidental damages plus the difference between the contract price and the salvage price. When a buyer breaches a contract to purchase by repudiating his offer, the seller is entitled to recover incidental damages plus either the difference between the contract price and the market price or the difference between the contract price and the resale price, less expenses saved as a result of the breach. Because the wholesaler chose to resell the strands of beads, his damages would be the difference between the resale (salvage) price and the contract price, plus incidental damages. Thus, the wholesaler can recover $9,000 less $4,000, plus incidental damages. (A) is incorrect because there is nothing in the UCC, which governs this contract, that makes contracts between merchants cancelable within a reasonable time. (B) is incorrect because the UCC remedy gives the seller the difference between the contract price and the resale price, not just the contract price. (D) is incorrect because it fails to account for the wholesaler's incidental damages.

Under the terms of a written contract, a contractor agreed to construct a garage for a homeowner for $10,000. Nothing was said in the parties' negotiations or in the contract about progress payments during the course of the work.After completing 25% of the garage strictly according to the homeowner's specifications, the contractor assigned his rights under the contract to a bank as security for an $8,000 loan. The bank immediately notified the homeowner of the assignment. The contractor thereafter, without legal excuse, abandoned the job before it was half-complete. The contractor subsequently defaulted on the loan from the bank. The contractor has no assets. It will cost the homeowner at least $8,000 to get the garage finished by another builder.If the bank sues the homeowner for $8,000, what will the court decide? A) The bank wins, because the contract between the contractor and the homeowner was in existence and the contractor was not in breach when the bank gave the homeowner notice of the assignment. B) The bank wins, because as a secured creditor the bank is entitled to priority over the homeowner's unsecured claim against the contractor. C) The homeowner wins, because his right to recoupment on account of the contractor's breach is available against the bank as the contractor's assignee. D) The homeowner wins, because his claim against the contractor arose prior to the contractor's default on his loan from the bank.

C) The homeowner wins, because his right to recoupment on account of the contractor's breach is available against the bank as the contractor's assignee. Answer C is correct. The bank, as the contractor's assignee of the contractor's rights under the contract between the contractor and the homeowner, is only entitled to the rights that the contractor would have, namely to collect the $10,000 upon completion of the contract in accordance with its terms. The contractor is in breach of the contract and entitled to nothing. Therefore, the bank as his assignee is entitled to nothing. Answer A is incorrect. The bank, as the contractor's assignee of the contractor's rights under the contract between the contractor and the homeowner, is only entitled to the rights that the contractor would have, namely to collect the $10,000 upon completion of the contract in accordance with its terms. The contractor is in breach of the contract and entitled to nothing. Therefore, the bank as his assignee is entitled to nothing. The fact that the contract was assigned at a time when the contractor was not in breach is irrelevant. Answer B is incorrect. As stated above, the bank is the assignee of the contractor's rights under the contract between the contractor and the homeowner. The bank is only entitled to the rights that the contractor would have, and the contractor is in breach of the contract. Answer D is incorrect. The issue is not when the respective rights under the contract arose. The bank's rights are derivative of the contractor's rights. Since the contractor is in breach and has no rights to damages against the homeowner, the bank also has no rights.

On April 1, the owner of a high-end boutique and a dress designer signed a written agreement wherein the boutique owner was appointed the "sole and exclusive" retail distributor for the designer's clothing in an upscale community. The contract provided that the designer was to have the absolute right to cease doing business with the boutique owner "at any time and on 30 days' notice." On May 1, the boutique owner handed the designer a written order for $50,000 worth of his original gowns. To publicize her new line of merchandise, the boutique owner conducted a large advertising campaign announcing to the community that she would have a wide selection of the designer's originals on display. She also made substantial improvements to the store to display these clothes in lush and expensive settings.On June 15, the designer faxed the boutique owner advising her that her competitor had made a proposal that he simply could not pass up, so he was invoking his right to terminate their arrangement on July 15. On receipt of the message, the boutique owner filed suit against the designer, seeking to prevent him from terminating the exclusivity agreement. In defense, the designer contended, among other things, that the April 1 exclusivity agreement with the boutique owner is unenforceable for lack of consideration, because the boutique owner did not obligate herself under the contract in any way.What is the best argument to rebut this contention? A) Any want of consideration at the formation stage was cured by the boutique owner's actual tender of a large order. B) The April agreement was, in effect, a "firm offer" between merchants and therefore binding on both parties even though there was no consideration. C) There was sufficient consideration in the boutique owner's implied promise to exercise her best efforts to promote the sale of the designer's gowns. D) The boutique owner conducted an advertising campaign and made substantial improvements to her store in reliance on the agreement.

C) There was sufficient consideration in the boutique owner's implied promise to exercise her best efforts to promote the sale of the designer's gowns. The best argument to rebut the designer's contention that there was not sufficient consideration in the boutique owner's promise is that the boutique owner impliedly agreed to use her best efforts to sell and promote the designer's gowns. For a contract to be valid, consideration must exist on both sides; i.e., promises must be mutually obligatory. Here, the designer has agreed to make the boutique owner his exclusive retail distributor, but the boutique owner's promise is less apparent—what is she obliged to do? The facts do not show any specific obligation. However, in a case where someone is to be the exclusive distributor, the court will imply a promise to use best efforts to sell the product. This implied promise is valid consideration both under the common law and the Uniform Commercial Code (applicable here because goods are involved). Thus, the contract between the boutique owner and the designer includes an implied promise that the boutique owner will use her best efforts to promote and sell the designer's gowns, and such a promise is valid consideration. (A) is wrong because merely placing an order for gowns would not cure the defect in the April 1 contract absent the boutique owner's implied promise, because the boutique owner still will not have promised to do anything under that contract (i.e., she is not bound to do anything referable to the April 1 contract); rather, all she has promised to do is to pay for the goods that she ordered, which is sufficient consideration only for the May 1 order, not for the entire contract. (B) is wrong because it does not address the issue raised by the boutique owner. A firm offer does not mean that the agreement is binding on both parties without consideration—it just means that a merchant can create an option-type offer without consideration for the holding open of the offer. (D) is wrong because the boutique owner's subsequent reliance on the agreement, while it may allow her to recover in restitution, does not address whether there was consideration at the formation stage, which is the issue raised by the designer here. Promissory estoppel (detrimental reliance) is an equitable doctrine used to enforce a promise when there is no consideration. Here, as discussed above, there was consideration on both sides.

The owner of a neighborhood tavern ordered 20 kegs of domestic beer from a brewery. The written contract between the parties provided that the brewery would deliver the beer by January 31, and the tavern owner would pay the brewery $1,400 upon delivery. The tavern owner was expecting a big crowd for Super Bowl Sunday, four days after the scheduled delivery date. The brewery did not deliver the beer to the tavern owner until February 16, but the tavern owner still accepted it.Did a duty to pay the $1,400 arise upon the tavern owner's acceptance of the beer on February 16? A) No, and the tavern owner retained the right to sue for any damages incurred because of the delay in delivery. B) No, but the tavern owner waived the right to sue for any damages incurred because of the delay in delivery. C) Yes, but the tavern owner retained the right to sue for any damages incurred because of the delay in delivery. D) Yes, and the tavern owner waived the right to sue for any damages incurred because of the delay in delivery.

C) Yes, but the tavern owner retained the right to sue for any damages incurred because of the delay in delivery. When the tavern owner accepted the beer on February 16, his duty to pay the $1,400 arose, but he retained the right to sue for damages because of the late delivery. The UCC requires perfect tender, which means that the goods and their delivery must conform to the contract in every way or there is a breach. Here, the brewery breached the contract by delivering the beer late. When a delivery is nonconforming, the buyer may: (i) reject the goods and cancel the contract or sue for damages; or (ii) accept any commercial units, reject the rest, and sue for damages. Once the goods are accepted, the buyer generally is bound on the contract and it is too late to cancel; however, the buyer retains the right to sue for damages for any nonconformity. When the tavern owner accepted the beer, he became bound under the contract to pay for the beer. Nevertheless, he can sue for damages that arose from the late delivery. (A) is incorrect because the duty to pay for the beer did arise when the tavern owner accepted the delivery. (B) is incorrect for two reasons: the duty to pay arose upon acceptance of the delivery, and the tavern owner did not waive his right to sue for damages. (D) is incorrect because the tavern owner retained the right to sue for damages despite accepting the delivery.

A builder of racing cars entered into a contract with a buyer to sell him a hand-built car for $25,000. The price was to be paid and the car was to be delivered one week later. Unbeknownst to the buyer, the builder's wife had a one-half interest in the car. The day after the contract was signed, the builder called the buyer and told him about his wife's half interest in the car, and that she would not go along with the sale at $25,000 but would agree to a sale for $40,000.If the buyer sues the builder for breach of contract, will the court find that there is an enforceable contract for $25,000? A) No, because the car cannot be sold unless both owners convey title. B) Yes, because the buyer was unaware of the interest of the builder's wife when he signed with the builder. C) Yes, regardless of the fact that the buyer was unaware of the interest of the builder's wife at the time he signed. D) Yes, but the contract is discharged by prospective inability of performance.

C) Yes, regardless of the fact that the buyer was unaware of the interest of the builder's wife at the time he signed. The builder's unconditional promise to sell created a contract even if the buyer knew of the builder's wife's interest. When a promise is unconditional, the failure to perform according to its terms is a breach of contract. By not making his promise conditional on his wife's consent to convey her interest, the builder impliedly undertook to obtain her consent. Therefore, the contract is enforceable. (Note that this does not necessarily mean that the buyer will be able to get the car; he may have to settle for damages because of the builder's wife's interest.) (A) is incorrect although partially true. It is true that the builder cannot sell his wife's half of the car without her consent; however, that does not make the contract here unenforceable. As stated above, by making his promise unconditional, the builder undertook a duty to obtain his wife's consent to sell the car. His failure to do so is a breach of contract, but a breach does not negate a contract; it merely gives the nonbreaching party a right to certain remedies. Therefore, the contract is enforceable, even if the builder's wife refuses to sell her interest. (B) is incorrect because the buyer's knowledge of the builder's wife's interest is irrelevant to the issue of the contract's enforceability. As discussed above, the builder's unconditional promise implied that the builder would obtain his wife's consent to convey her interest in the car. Therefore, the contract is enforceable regardless of whether the buyer was aware of the builder's wife's interest at the time he signed. (D) is incorrect because "prospective inability" is not a ground for discharge. Prospective failure of consideration occurs when a party has reasonable grounds to believe that the other party will be unable or unwilling to perform when due. The prospective inability of performance does not discharge the contract; rather, it allows the innocent party to suspend further performance until he receives adequate assurances that performance will be forthcoming. Therefore, the contract between the builder and the buyer is not discharged because of prospective inability of performance.

Bennett, an aspiring writer, has written his first novel, "Birds in the Bush, But None in the Hand". Chandra is a literary agent. The parties form a contract under which Chandra, for one year, will seek publishers for Bennett's novel, with Bennett to pay her 5% of all royalties the novel might earn if published. Chandra never does make an effort to secure any such publishers; she breaches her contract. Bennett visits his attorney and asks, "Can I sue Chandra?" The lawyer's answer should be: Question 6 options: A) "Yes, absolutely." B) "You might or might not have a legitimate action depending on the court's opinion." C) "You have a right to sue anyone for anything, but that does not mean you'll win." D) "You have a legitimate cause of action, but you will not likely recover any meaningful award."

D) "You have a legitimate cause of action, but you will not likely recover any meaningful award." D is correct answer here. Bennett promised to pay Chandra 5% of all royalties, but only if his novel is published. Under the contract, he is not paying her for any services until she locates a publisher to publish the novel. Bennett does not sustain any expenses in reliance on the contract either. So, if Chandra does not do anything, even if she is acting in bad faith, Bennett arguably suffers no loss. If so, although he has a legitimate cause of action for breach of contract, he will likely recover no meaningful amount of damages (e.g., he might get nominal damages, see Restatement (2nd) of Contracts, Sec. 346(2))

On April 1, a music store owner offered to sell a rare piano to his best customer, a concert pianist, for $100,000. The following day, the pianist, who performs around the world with two of the several pianos he has purchased from the store, wrote to the store owner: "I have decided to purchase the piano. A check for $100,000 is enclosed. I am leaving in one week for Canada. I will be gone for one month and will pick up the piano when I return. I will pay you to store the piano in your air-conditioned warehouse." One week later, the pianist left for Canada without hearing from the music store owner. What does the letter that the pianist wrote to the store owner constitute? Question 8 options: A) A conditional acceptance. B) A rejection of the offer. C) An acceptance, and the store owner is not bound to store the piano. D) An acceptance, and the store owner must store the piano but is entitled to the reasonable value of that service.

D) An acceptance, and the store owner must store the piano but is entitled to the reasonable value of that service. Answer (D) is correct. The pianist's letter to the store owner is an acceptance, and the store owner must store the piano. This is a contract for a sale of goods and thus is governed by the UCC. Under the UCC, an acceptance with additional terms does not constitute a rejection and counteroffer, but rather is an effective acceptance unless made expressly conditional on the assent to the additional terms. Here, the pianist expressly accepted the store owner's offer and included an additional term adding one month of storage. The acceptance was not expressly conditional on the store owner's assent to the storage term. Thus, the acceptance was sufficient to create a contract. Whether additional terms become part of the agreement depends on whether both parties are merchants. If both parties are merchants, the additional terms become part of the contract unless they materially alter the terms of the offer, the offer expressly limited the acceptance to its terms, or they are objected to within a reasonable time. A "merchant" is one who regularly deals in goods of the kind sold or who otherwise by his occupation holds himself out as having knowledge or skill peculiar to the practices or goods involved. For purposes of the UCC battle of the forms provisions, a merchant is almost anyone in business because anyone in business has knowledge of business practices. Here, the store owner is clearly a merchant. The pianist, by virtue of his occupation, has knowledge or skill peculiar to the goods (piano) involved. He plays the piano professionally and has purchased several pianos for this purpose. The additional term included in the pianist's acceptance did not materially alter the terms of the offer (i.e., it did not change a party's risk or remedies), the offer was not limited to its terms, and the facts indicate that the store owner did not object. Thus, the additional term regarding storage becomes part of the agreement.

A landowner entered into a written agreement with an architect. Under the agreement, the architect agreed to design a vacation home to be built on the landowner's property, and, upon the landowner's approval of the design plan, to supervise construction of the home on the property. The agreement was silent as to assignment or delegation by either party.Before beginning the work, the architect sold his business to an investor under an agreement in which the architect assigned to the investor, and the investor agreed to complete, the contract between the landowner and the architect. The investor, also an experienced architect of excellent repute, advised the landowner of the assignment, and supplied him with information confirming both the investor's financial responsibility and past commercial success.The landowner allowed the investor to perform and approved the investor's design plan, but the investor failed without legal excuse to supervise the construction of the home as agreed.Against whom does the landowner have an enforceable claim for breach of contract? A) The architect only, because the architect's agreement with the investor did not discharge his duty to the landowner, and the investor made no express promise to the landowner. B) The investor only, because the architect's duty to the landowner was discharged when the architect obtained a skilled architect (the investor) to perform the contract between the landowner and the architect. C) The investor only, because the landowner was an intended beneficiary of the agreement between the architect and the investor, and the architect's duty to the landowner was discharged when the landowner permitted the investor to do the work and approved the investor's design. D) Either the architect because his agreement with the investor did not discharge his duty to the landowner, or the investor because the landowner was an intended beneficiary of the agreement between the architect and the investor.

D) Either the architect because his agreement with the investor did not discharge his duty to the landowner, or the investor because the landowner was an intended beneficiary of the agreement between the architect and the investor. Answer D is correct. Unless there is an express novation, a delegator (the architect) remains liable for failures of the delegate (the investor). Furthermore, a delegate will be liable to an obligee unless the delegate is merely an employee of the delegator (the original obligor). In addition, since the architect and the investor entered into a contract that provided specifically for the investor to satisfy the architect's obligations to the landowner, the landowner was an intended beneficiary of the agreement between the architect and the investor, and as such is able to bring an action personally for the investor's breach of that contract. Answer A is incorrect. No direct promise need be made to an intended third-party beneficiary for that beneficiary to successfully enforce that contract. Answer B is incorrect. Merely finding a skilled person to perform does not release a delegator. The facts do not state that the landowner released the architect—a key element of a novation. Without that release, both the delegator and the delegate are liable if there is a breach, no matter how careful a choice is made by the delegator in choosing a delegate. Answer C is incorrect. The facts do not state that the landowner released the architect—a key element of a novation. Without that release, both the delegator and the delegate are liable if there is a breach, even with an affirmative acceptance of the delegate by the obligee.

In a Cleveland city newspaper Arlene posts this advertisement: "Leather handbag lost on downtown city bus; $100 reward for its return [address, telephone number, email address]." Jack lives in Cleveland, but has not seen Arlene's advertisement. Riding the downtown city bus, he finds the handbag. In it, he discovers a writing that shows Arlene's home address, to which he then travels. Finding no one at the home, he leaves the handbag in Arlene's mailbox. Is Jack entitled to the $100 reward? Question 3 options: A) Yes, because the advertisement was an offer for a unilateral contract. B) Yes, because he lived in the city within which the newspaper circulated. C) No, because he failed to deliver the handbag personally to Arlene. D) No, because he acted without knowledge of the advertisement.

D) No, because he acted without knowledge of the advertisement. Answer D is correct here. To have the power to accept, the offeree must have knowledge of the offer. Therefore, the proposal must be communicated to the offeree. Here, Jack was unaware of the advertisement (promise of the reward). Hence, he is not entitled to the $100 reward. See BarBri outline for the rules on "The Offer"-> "Communication to the Offeree." An example that BarBri gives: Chauncey returned Bowater's lost briefcase unaware that Bowater had placed an advertisement offering a $20 reward for its return. Because the offer had not been communicated to Chauncey, there could not be mutual assent. Hence, there is no contract.

A homeowner contracted with a local builder to build a wooden deck onto the back of her house. The contract called for half of the contract price of $20,000 to be paid to the contractor before he began work and the other half to be paid to him when the job was completed. The contractor began the work but, partway through the job, he got an offer for a rush job that paid better and abruptly quit. The homeowner sues the contractor for specific performance. Will she prevail? A) Yes, because there has been a novation. B) Yes, because the contract between the parties was valid and the contractor had no legal justification for abruptly quitting. C) No, because by not paying the contractor for the second half of the job, the homeowner has not satisfied all of her conditions under the contract. D) No, because the contract is for personal services.

D) No, because the contract is for personal services. Rationale The homeowner will not prevail in her suit for specific performance because the contract is for personal services. Specific performance is available only where the legal remedy (i.e., money damages) is inadequate. Money damages can be inadequate for a number of reasons, such as where the goods or services sought are unique. Nevertheless, specific performance is not available as a remedy regarding a contract for services. If the services contracted for are not unique, then money damages would be adequate because the nonbreaching party could use the damages to hire someone else to perform. However, even if the services are unique, courts generally will not grant a decree of specific performance to force someone to work because of problems in overseeing performance and because it would be tantamount to involuntary servitude. (A) is wrong because a novation is a substitution of a third party for one of the parties to a contract by agreement of all the parties involved, and nothing in the facts indicates that such a substitution was involved here. Moreover, if a novation were involved, the homeowner would not be able to force the contractor to work because the contractor would have been the party released. (B) is wrong because a valid contract and no legal justification for breaching it do not, by themselves, mean that specific performance is available, as discussed above. (C) is wrong because, under the contract, the homeowner was not required to make the second half of the payment to the contractor until the contractor's work was completed. One of the prerequisites for specific performance is that all of the plaintiff's contractual conditions have been fulfilled. This includes the fulfillment of all conditions precedent and a readiness to perform any conditions concurrent. Here, the homeowner stood ready and able to perform under the contract when the contractor performs, so the homeowner's failure to pay the contractor for the second half of the job would not be a bar to specific performance.

The owner of a stationary bicycle wrote a letter to her friend offering to sell her stationary bicycle to him for $150. The friend received the letter on January 18. On January 19, he mailed a letter back saying that he was not interested in purchasing the bike because he had just purchased a gym membership. However, the friend changed his mind the next day and mailed a letter to the owner accepting her offer to sell the bicycle and enclosing a certified check for $150. The owner received the friend's rejection letter on January 21 but put it aside without reading it. The next day, she received the friend's acceptance letter, which she opened and read immediately.Do the parties have a contract? A) Yes, because under the mailbox rule an acceptance is effective on dispatch, while a rejection is effective on receipt. B) Yes, because the friend paid for the bicycle when he accepted the offer to buy it. C) No, because the acceptance was dispatched after the rejection. D) No, because the mailbox rule does not apply—whichever is received first controls.

D) No, because the mailbox rule does not apply—whichever is received first controls. The parties do not have a contract, because the mailbox rule does not apply when the offeree sends a rejection, followed by an acceptance. In such a case, whichever is received first controls. Under the mailbox rule, acceptance by mail or similar means creates a contract at the moment of posting, with a couple of exceptions not relevant here. Rejection, on the other hand, is effective when received. So, if the mailbox rule had applied, there would have been a contract, because the friend's acceptance was mailed before his rejection letter was received. But because the mailbox rule does not apply here, and the matter is decided based on which letter was received first, there is no contract, because the friend's rejection letter was received by the bicycle owner a day before his acceptance letter was received by her. (A) is incorrect because, as discussed above, the mailbox rule does not apply when a rejection is sent before an acceptance; rather, whichever is received first controls. The fact that the bicycle owner did not read the rejection does not matter; it still was received by her before the acceptance. [See Restatement (Second) Contracts §68] (B) is incorrect because whether the friend paid for the bicycle is irrelevant. He sent the certified check (and his acceptance) after he sent his rejection, and the rejection was received first. (C) is incorrect because when a rejection by mail is followed by an acceptance by mail, the rule is that whichever is received first controls, not whichever is dispatched first. Thus, although it is true that there is no contract between the parties, it is because the friend's rejection letter was received by the bicycle owner first, rather than because it was mailed first.

A cat lover owned a cat of little value. One day, the cat disappeared, and the owner placed the following advertisement in the newspaper: "I will pay $500 to anyone who returns my cat." A mail carrier found the cat the next day a few blocks away. Knowing that the cat belonged to the cat lover, the mail carrier took the cat home, intending to return him the next day on her route. The mail carrier telephoned the cat lover as soon as she got home, before reading the advertisement. Before the mail carrier could tell the cat lover that she had the cat, the cat lover said, "I did a foolish thing in offering a reward for my cat. I've changed my mind, and I'm going to cancel the reward by putting a new advertisement in the paper tomorrow." Can the mail carrier return the cat and collect the reward? Question 1 options: A) Yes, the offer can be accepted until a revocation is published in the newspaper. B) Yes, because she accepted the offer by finding the cat and calling the cat lover. C) Yes, because the offer became irrevocable because the mail carrier began to perform the requested act. D) No, the mail carrier cannot accept the offer because the offer was revoked.

D) No, the mail carrier cannot accept the offer because the offer was revoked. Answer D is correct. Although the general rule is that an offer made by publication may be revoked only by publication through comparable means, an offer may always be revoked by direct communication to the offeree. Here, the mail carrier had direct notice of the cat lover's revocation of the reward offer. As to the mail carrier, the offer was revoked at the time she received that notice; as to the general public, the offer will be revoked when the revocation is published. The revocation terminates the offer, and the mail carrier can no longer accept it. Answer A is incorrect. A revocation is effective at the time an offeree has actual or constructive notice or knowledge of it. When the mail carrier spoke with the cat lover, she received actual notice that the reward offer was revoked, and so she could no longer accept it. Although a general offer such as the one made by the cat lover can only be revoked as to the general public by a notice given publicity equal to that given the offer, personal notice of the offeror's intent to revoke is also effective as against any individual offeree. Answer B is incorrect. The reward was an offer for a unilateral contract. An offer for a unilateral contract is accepted only when the performance requested is completed. Here, the performance is not complete until the cat is returned. Thus, the mail carrier did not accept the offer when she found the cat and called the cat lover. Although an offer for a unilateral contract becomes irrevocable when the offeree begins performance in reliance on the offer, here, the mail carrier did not know of the offer and thus was not acting in reliance on it when she found the cat and called the cat lover. Since the offer had not become irrevocable, the cat lover was free to revoke it. When the mail carrier spoke to the cat lover on the telephone, she received actual notice that the offer was revoked, and she could no longer accept it. Answer C is incorrect. The reward was an offer for a unilateral contract. An offer for a unilateral contract becomes irrevocable when the offeree begins performance in reliance on the offer. Here, the mail carrier did not know of the offer and thus was not acting in reliance on it when she found the cat and called the cat lover. Since the offer had not become irrevocable, the cat lover was free to revoke it. When the mail carrier spoke to the cat lover on the telephone, she received actual notice that the offer was revoked, and she could no longer accept it.

A psychotherapist group hired a construction company to construct a one-story building for $900,000 to use as the group's offices. The contract provided that the group would pay one-third of the contract price (i.e., $300,000) on completion of the foundation, another one-third ($300,000) on completion of the roof, and the balance ($300,000) when the rooms were carpeted. After completing work on the foundation, the construction company abandoned the project. The group hired another contractor to complete the building. How much, if anything, can the construction company recover? A) $300,000, the contract price for the work that it completed. B) $300,000, offset by whatever damages the group suffered in getting the job completed by someone else. C) Nothing, because the construction company committed a material breach of the contract. D) Nothing on the contract, because this construction contract is not divisible in nature, but it might be entitled to restitution.

D) Nothing on the contract, because this construction contract is not divisible in nature, but it might be entitled to restitution. The construction company cannot recover on the contract but it may recover restitution for the work it did. The issue here is whether this contract is divisible. A contract is divisible if: (i) the performance of each party is divided into two or more parts under the contract, (ii) the number of parts due from each party is the same, and (iii) the performance of each part by one party is the agreed-upon equivalent of the corresponding part from the other party. The third requirement is the problem here. There is no indication that the parts the construction company is to perform are the equivalent of the payments the group is to make. Rather, these payments appear to be unrelated to the actual work and merely represent progress payments. Thus, this contract is not divisible in nature. Because the construction company breached the contract by ceasing performance before completion of the entire job, it is not entitled to the contract price. However, it may be entitled to restitution. Modern courts permit a breaching party to recover restitution where the party has conferred a benefit on the other that will result in unjust enrichment of the other if no compensation is paid. Here, the construction company properly completed the foundation, and this conferred a benefit on the group. Therefore, restitution may be available to the construction company. (A) is wrong because, as mentioned, the construction company is not entitled to the contract price, because this is not a divisible contract. Even if it were divisible, (A) would be wrong because the group would be able to offset its damages for the construction company's breach and thus would not be obliged to pay the full contract price. (B) would be the correct answer if the contract were divisible. As explained above, this contract is not divisible merely because it calls for three payments. The payments must be the equivalent of the work performed; otherwise the payments are mere progress payments and the times set for payment are simply convenient times to pay. (C) is wrong because although the breach was material, the construction company could recover something in restitution (see above).

An owner of land who was also a home contractor agreed in writing with a buyer to build a house to the buyer's specifications on his (the contractor's) land and then sell the house and lot to the buyer. The contract provided that the house was to be completed by March 1, with full payment due at that time. On March 1, the house was nearly complete, but due to delays in the delivery of materials, the contractor would need 20 more days to finish construction. On March 5, after discovering that the house had not yet been completed, the buyer notified the contractor in writing of her election to cancel the contract because of the contractor's failure to deliver the house by March 1. The contractor responded that, due to an unanticipated strike at his supplier's company, performance had been unforeseeably delayed and that the house would be ready by March 20. The buyer responded that she would no longer accept delivery of the house and land. The contractor then brought an action to recover damages for breach of contract.Who will prevail? A) The buyer, because the express date listed in the contract indicates that time was of the essence. B) The buyer, because delivery by March 1 was a condition precedent to the buyer's performance. C) The contractor, because the strike was truly an unforeseeable intervening event. D) The contractor, because the buyer received the substantial benefit of the bargain.

D) The contractor, because the buyer received the substantial benefit of the bargain. The contractor will prevail because the buyer received the substantial benefit of her bargain. The failure to perform on time is a breach of contract, but in this case, it was a minor breach. Unless the nature of the contract is such as to make performance on the exact day agreed upon of vital importance, or the contract provides that time is of the essence, failure to perform at the stated time is not a material breach. Here, the home was nearly complete, and the delay was relatively short. The contract did not specify that time was of the essence; thus, the breach was minor. The remedy for a minor breach is damages; the aggrieved party is not relieved of her duty to perform. (A) is incorrect because merely stating a date for performance does not indicate that time is of the essence. There must be some explicit statement indicating that time is of the essence. (B) is incorrect. Although the delivery on March 1 is a condition precedent to the buyer's duty to pay, the condition is excused by substantial performance. The test for whether a party has substantially performed is the same as the one for assessing whether a breach is minor or material. Here the breach is minor, the contractor substantially performed, and the condition is excused. (C) is incorrect because an unforeseeable event does not discharge a party's duty to perform. A strike at the contractor's supplier does not rise to the level of impossibility or impracticability, which would discharge his duty to perform. The contractor could have procured the supplies elsewhere.

A bathroom fixture company, in a signed writing, contracted with the developer of an apartment building for the sale to the building of 50 identical sets of specified bathroom fixtures, 25 sets to be delivered on March 1, and the remaining 25 sets on April 1. The agreement did not specify the place of delivery, nor the time or place of payment. On March 1, the fixture company tendered 24 sets to the developer and explained that one of the 25 sets was damaged in transit from the manufacturer to the company. The fixture company also promised to deliver a replacement within five days.Which of the following statements is correct? A) The developer is entitled to accept any number of the 24 sets, reject the rest, and cancel the contract both as to any rejected sets and the lot due on April 1. B) The developer is entitled to accept any number of the 24 sets and to reject the rest, but is not entitled to cancel the contract as to any rejected sets or the lot due on April 1. C) The developer must accept the 24 sets but is entitled to cancel the rest of the contract. D) The developer must accept the 24 sets and is not entitled to cancel the rest of the contract.

D) The developer must accept the 24 sets and is not entitled to cancel the rest of the contract. The developer must accept the 24 sets and is not entitled to cancel the rest of the contract. The ground for rejection of an installment contract is UCC section 2-612(2), which provides that the buyer may reject any installment that is nonconforming only if it substantially impairs the value of that installment and cannot be cured. If the nonconformity can be cured and the seller gives adequate assurances of its cure, the shipment must be accepted by the buyer, provided that the defect is not such as to constitute a breach of the whole contract. (A) is incorrect. Under UCC section 2-601, if defective goods arrive, the buyer normally has every right to reject them as unsatisfactory. But this is not the case if the defective shipment was part of an installment contract, as is the case here. (B) is incorrect for the reasons stated above. (C) is incorrect. Under UCC section 2-612(3), there is a breach of the entire contract only when the nonconformity or default with respect to a single installment substantially impairs the value of the whole contract. Here, this is clearly not so, and the developer may not repudiate the second shipment based upon the single incomplete set.

Neighbors of an apparently destitute couple bought a month's supply of food and gave it to them. Later, the wife confided in the neighbors that she and her husband did have money and that, because they had been so kind, she was leaving them money in her will. When the wife died, at the neighbors' request the husband gave the neighbors the following signed instrument: "In consideration of my wife's promise to our neighbors, and their agreement not to sue her estate, I agree to pay them the sum of $5,000." When the husband died of a heart attack several days later, the neighbors asked the administrator of his estate to pay them the $5,000. The administrator refused on the ground that there was no consideration for the agreement.On which of the following theories would it be most likely that the neighbors would recover? A) The husband's written instrument was a binding unilateral contract. B) The husband's acceptance of the food was fraudulent. C) The husband is bound by promissory estoppel. D) The husband and the neighbors entered into a valid compromise.

D) The husband and the neighbors entered into a valid compromise. The most likely theory is that the husband and the neighbors entered into a valid compromise. If the neighbors have given up a good faith claim, their agreement with the husband is a compromise supported by valid consideration. Thus, there is an enforceable contract. (A) is wrong because the husband was requesting a promise from the neighbors, not an act. (B) is wrong because the husband was under no obligation to reject a gift if he did not deliberately induce the neighbors to give it. (C) is wrong because there is no evidence that the neighbors gave the food in reliance on any promise made by the husband.

A wholesale seller sent a fax to a manufacturer with whom he had done business before: "Send 500 'Madewell' chairs at your usual price." The manufacturer responded, also by fax, that the line was being discontinued, but he would ship his last 500 chairs at $75 per chair, his usual price. The manufacturer immediately began the paperwork for processing the order and started preparing and packing the chairs for shipment. Before the chairs could be delivered, the wholesaler canceled his order, noting that the price was too high. The day after receiving the wholesaler's cancellation, the manufacturer sold the chairs to another buyer for $75 each. If the manufacturer sues the wholesaler for damages, how much should he recover? A) Nothing, because this was a contract between merchants and the wholesaler canceled within a reasonable time. B) Nothing, because the manufacturer was able to cover by selling the chairs at the same price he would have received from the wholesaler. C) $37,500, the full contract price, because the wholesaler breached the contract and $75 per chair was a fair price. D) The incidental costs of preparing the paperwork and other office costs connected with preparing and packing the chairs for shipment to the wholesaler.

D) The incidental costs of preparing the paperwork and other office costs connected with preparing and packing the chairs for shipment to the wholesaler. The manufacturer will recover only his incidental damages, i.e., the costs of preparing to ship the chairs. An offer calling for shipment of goods, such as the offer here, may be accepted by prompt shipment with notice or by a promise to ship. Acceptance forms a contract. Here, the manufacturer accepted the wholesaler's offer by promising to ship, and a contract was formed. The wholesaler breached the contract by canceling his order. When a buyer breaches by repudiating his offer, as the wholesaler did here, the seller has a right to recover his incidental damages plus either the difference between the contract price and the market price or the difference between the contract price and the resale price of the goods, reduced in either case by any expenses saved as a result of the breach. Here, the manufacturer made what he would have if the sale with the wholesaler had gone through—there was no difference between the contract price and the resale price. Thus, the manufacturer would be limited to his incidental damages. Lost profits would not be available because the chairs were the last ones that the manufacturer had and would have because the line was being discontinued. Therefore, the manufacturer would not have been able to sell another set of 500 chairs to another potential buyer. (A) is incorrect because there is no rule under the UCC, which governs the contract here, that makes contracts between merchants cancelable within a reasonable time. (B) is incorrect because, as indicated above, the UCC allows the seller to recover incidental damages. (C) is incorrect because the UCC seeks only to put the nonbreaching party in as good a position as it would have been in had the other party performed. Here, awarding the manufacturer the full contract price would put him in a better position than performance would have, because it would give him a double recovery for selling the same goods. (The result would be different, however, if the manufacturer had had more chairs to sell, because in that case, the breach would have cost the manufacturer additional sales—i.e., he could have sold to the wholesale seller and to the second buyer.)

An artist, a baker, and a cook are identical 70-year-old triplets who have been very close during their lives. The artist and her husband are retired and living on a modest fixed income. The baker is a successful business executive. The cook has suffered a slight stroke and is living in a nursing home in a city about 200 miles from the artist's home.One day, the artist's husband telephoned the baker and told her that he had just ordered a $2,000 diamond ring as a Valentine's Day present for his wife. When the baker heard about the husband's proposed present, she told him, "It must be hard for you to pay for that ring on your retirement income. If you promise to bring my sister to visit our other sister in the nursing home on our birthday next October, I will pay the jeweler for the ring." The husband said: "That will be wonderful. I'll be delighted to take you up on your offer."The husband picked up the ring, gave it to his wife, and told the jeweler to send the bill to the baker. The baker then suffered a severe stroke, and the conservator appointed to handle her affairs refused to pay the bill sent by the jeweler.If the jeweler sues the baker's conservator in May for $2,000, who will prevail? A) The conservator will prevail, because the husband has not performed the consideration required. B) The conservator will prevail, because of the Statute of Frauds. C) The jeweler will prevail, because the main purpose of the baker's request was for her own benefit. D) The jeweler will prevail, because it was the intended beneficiary of the baker's promise.

D) The jeweler will prevail, because it was the intended beneficiary of the baker's promise. Answer D is correct. The facts present a valid third-party beneficiary contract where the jeweler is the intended beneficiary. A third-party beneficiary may enforce the contract, subject to any defenses that the promisor has against the promisee, such as lack of assent, lack of consideration, illegality, impossibility, and failure of a condition. There is no such defense here. The husband's promise to bring the artist to the nursing home for a visit is good consideration to support the promise. The husband is not in breach of the contract, because the time to perform has not yet come. Therefore, the baker's conservator cannot claim that the husband's breach justifies her refusal to perform. Answer A is incorrect. The jeweler is the third-party beneficiary of a valid contract between the husband and the baker. The husband's promise to bring the artist to the nursing home for a visit is good consideration to support the promise even though the consideration is not being given to the baker directly. The husband is not in breach of the contract, because the time to perform has not yet come. Therefore, the conservator cannot claim that the husband's breach justifies her refusal to perform. Answer B is incorrect. The Statute of Frauds is inapplicable. The baker is not promising to guaranty the debt of another, but rather has assumed the primary responsibility for that debt as the result of a bargain made with the husband. The baker agreed to pay for the ring the husband ordered if the husband agreed to bring his wife to the nursing home on a specific date; she did not agree to pay for the ring if the husband could not do so as would be the case in a suretyship promise. In addition, the baker's promise is not for the sale of goods; it is merely to pay the bill owed by the husband. Answer C is incorrect. This response contains language from an exception to the suretyship provision of the Statute of Frauds— the main purpose rule. Under that rule, if the main purpose of the promise is to serve a pecuniary interest of the promisor, the contract is not within the Statute of Frauds even though the effect is still to pay the debt of another. But, the suretyship provision of the Statute of Frauds is inapplicable here, because the baker is not promising to guaranty the debt of another, but rather to assume primary responsibility for the husband's debt as the result of a bargain made with the husband.

large appliance store entered into a written agreement with a television manufacturer for 20 high definition televisions for $300 each. The shipment arrived on time and was paid for with a cashier's check by the appliance store. However, when the appliance store tested the TVs, it found that none of them met the high definition specifications in the contract, even though they functioned fine otherwise. The appliance store notified the manufacturer that it was rejecting the TVs because they were nonconforming, but the manufacturer did not want them returned.What may the appliance store recover? A) Nothing. B) The money it has already paid for the TVs, and it gets to keep them. C) The difference between the value of the TVs and the price it could have obtained for them on the market. D) The money it paid for the TVs, but it may sell them for the manufacturer's account.

D) The money it paid for the TVs, but it may sell them for the manufacturer's account. The appliance store may recover the money it paid for the 20 TVs and resell them for the TV manufacturer's account. A buyer who has rejected goods as nonconforming is entitled to any prepayment, or, if the seller refuses to refund, to resell the goods and apply the proceeds to what is owed him from the seller. The buyer is entitled to offset expenses of selling and get its money back. Hence, (D) is correct and (A) is wrong. The buyer is not entitled to keep the TVs and get its money back. Thus, (B) is wrong. (C) is wrong. Had the appliance store accepted the 20 nonconforming TVs, it would be entitled to the difference between the value of the TVs and the value they would have had if they had conformed to the contract. Here, however, the appliance store rejected the TVs.

On May 15, a pillow manufacturer e-mailed an order to a merchant who sells feathers: "Ordering 6,000 pounds of feathers at $2 per pound." When the order was placed, the manufacturer expected the feathers to be delivered on June 1, because the merchant typically made deliveries on the first of the month. The feather merchant replied that he would fill the order but added that he would have 4,000 pounds ready on June 1, with another 2,000 pounds ready June 8. The pillow manufacturer did not respond. What is the effect of the communications between the pillow manufacturer and the feather merchant? A) No contract was formed, because the pillow manufacturer did not assent to the proposed delivery schedule within a reasonable time. B) No contract was formed, because the feather merchant's terms constituted a rejection of the pillow manufacturer's offer. C) There is a contract for the delivery of 6,000 pounds on June 1. D) There is a contract for delivery of 4,000 pounds on June 1 and 2,000 pounds on June 8.

D) There is a contract for delivery of 4,000 pounds on June 1 and 2,000 pounds on June 8. Answer D is correct. This question deals with a transaction for the sale of goods between merchants and is governed by Article 2 of the UCC. Article 2 allows a definite and seasonable expression of acceptance to operate as an acceptance, even though it states additional or different terms to the original offer. When, as here, both parties are merchants, the additional terms become part of the contract, unless the offer limits acceptance to the terms of the offer, the additional terms materially alter the offer, or an objection to the additional terms is seasonably given. Since none of those conditions are indicated by the facts, the additional delivery terms became part of the contract, and Answer D is correct, because it incorporates the feather merchant's shipping dates. Answer A is incorrect. The feather merchant made a definite and seasonable expression of acceptance by stating that he would fill the order. Although the feather merchant's acceptance did vary the delivery terms, it did not expressly state that it was conditional upon the pillow manufacturer's assent to the additional delivery terms. Therefore, the pillow manufacturer's assent was not required for contract formation, contrary to the statement made in this answer choice. Also, this is not a sufficiently different term to be deemed a material alteration. Since both parties are merchants, the additional terms would become part of the contract, unless the pillow manufacturer objected to them. Answer B is incorrect. At common law, the feather merchant's acceptance varying the terms of the offer would operate as a rejection of the pillow manufacturer's offer and a counteroffer; no contract would be formed unless the pillow manufacturer accepted the counteroffer. However, this is a contract for the sale of goods and is governed by UCC Article 2. As explained above, under Article 2, an acceptance that contains new or different terms is still effective as an acceptance. Since both parties here are merchants, not only is the acceptance effective but the new terms become part of the contract. Answer C is incorrect. As explained above, since both parties are merchants, the new delivery terms become part of the contract, because the pillow manufacturer has not objected to them, they do not materially alter the contract, and the offer does not limit acceptance to the terms of the offer. Therefore, Answer C is incorrect, because it does not recognize the new delivery terms.

A third year law student fell behind in paying his rent and was facing eviction. He felt he had no choice but to drop out of law school and get a full-time job. The student's father wanted his son to graduate, so they struck a deal. The father would pay his son's back rent and his rent for the remainder of the school year if the student agreed to stay enrolled in school. The father wrote a letter to his son's landlord outlining their agreement. The landlord did not reply to the father's letter, but he did not evict the law student. The student stayed in school and graduated. The following day, the father died suddenly without making any payments to the landlord. The landlord subsequently sued the father's estate as the third-party beneficiary of the contract between the law student and his father.Will the landlord be successful in this claim? A) No, because the contract between the father and the student was not supported by consideration. B) No, because the landlord was only an incidental beneficiary to the contract between the father and the student. C) Yes, because the letter provided the landlord with notice of the contract between the father and the student and the landlord relied upon it. D) Yes, because the law student did not drop out of law school.

D) Yes, because the law student did not drop out of law school. Answer D is correct. The contract between the law student and his father, whereby the father promised to pay the law student's rent to the landlord in exchange for the law student's promise to stay enrolled in school, was an enforceable third-party beneficiary contract in which the landlord was the intended beneficiary. The contract has not been rescinded by the original contracting parties, and the law student has performed his obligation under the contract. His performance of that obligation makes the father's promise to pay the landlord enforceable. Answer A is incorrect. The contract between the law student and his father was an enforceable third-party beneficiary contract in which the landlord was the intended beneficiary. The contract was supported by adequate consideration in that the law student was promising to stay enrolled in school —something that he was not otherwise obligated to do—in exchange for his father's agreement to pay the rent. Consideration is adequate even if it does not directly benefit the party requesting it, as long as it is bargained for. Answer B is incorrect. The parties to the contract, the law student and his father, intended to benefit the landlord by the father's paying the law student's rent. Since the whole point of the contract between the law student and the father was to ensure that the landlord would be paid the rent, the landlord is an intended beneficiary of their contract and not an incidental beneficiary. As an intended beneficiary, the landlord has a right to sue on the contract. Answer C is incorrect because it states the wrong reasoning. The rule about notice relates to the vesting of contractual rights in the third party such that the contracting parties cannot modify or extinguish the third party's rights under the contract. Here, the contracting parties are not attempting to modify the contract. The law student has fully performed the contract and the landlord now merely seeks performance of the other side's obligation. The father's estate now owes performance to the landlord because the law student has performed his obligation under the contract.

A buyer mailed a signed order to a seller that read: "Please ship us 10,000 widgets at your current price." The seller received the order on January 7, and that same day mailed to the buyer a properly stamped, addressed, and signed letter, accepting the order at the seller's current price of $10 per widget. On January 8, before receipt of the seller's letter, the buyer telephoned the seller and said, "I hereby revoke my order." The seller protested to no avail. The buyer received the seller's letter on January 9. As of January 10, was there a contract between the buyer and the seller? A) No, because orders are offers that can be accepted only by shipping the goods, and the offer was revoked before shipment. B) No, because the buyer never agreed to the $10 price term. C) Yes, because the order was, for a reasonable time, an irrevocable offer. D) Yes, because the seller accepted the offer before the buyer attempted to revoke it.

D) Yes, because the seller accepted the offer before the buyer attempted to revoke it. Answer D is correct. The buyer sent an offer, which the seller accepted by properly mailing the January 7 response. Under the mailbox rule, unless the offeree specifies otherwise, or the offer is covered by an option contract, an acceptance is effective upon dispatch; i.e., when the offeree properly posts his letter of acceptance. Thus, the contract here was created on January 7 when the seller mailed the acceptance. The buyer's attempt to revoke the offer on January 8 was completely ineffective because the acceptance terminated the buyer's power of revocation. Answer A is incorrect. Orders for the shipment of goods may be accepted either by prompt shipment or the promise to ship. Here, the order was effectively accepted by the seller's promise to ship the goods. Answer B is incorrect. The offer established the price at the seller's current price on January 7, which was $10. Answer C is incorrect. Unless there is an option contract (involving consideration given to keep an offer open) or a merchant's firm offer (a promise by a merchant to hold the offer open for a specified period), an offer may be revoked at any time, for any reason, prior to acceptance.

A large oil producer wanted to expand into the oil refinery business. It contacted a small oil refinery company and offered to purchase the company's refining facilities as well as its output of petroleum products from its remaining production facilities.Lawyers for the oil producer prepared a written agreement specifying the sale price for the smaller company's refining business. The agreement also included provisions permitting the oil producer, at its option, to purchase 100% of all petroleum products removed from the earth by the smaller company for the following 15 years, at current market prices. The agreement was signed by both parties. One month after the sale of the refining facilities, the oil producer sent a fax to the smaller company stating that it would be exercising its option to purchase the smaller company's oil output for the next 15 years.Is the written agreement between the oil producer and the smaller company enforceable? A) No, because the provision permitting the oil producer to purchase all of the smaller company's output was not supported by any consideration. B) No, because the provision permitting the oil producer to purchase all of the smaller company's output for 15 years was indefinite and uncertain. C) Yes, as to the sale of the smaller company's refining facilities, but unenforceable as to the output provision. D) Yes, in its entirety.

D) Yes, in its entirety. The written agreement is enforceable in its entirety. The initial contract had two components: the sale of the smaller company's refining facilities to the oil producer, and the oil producer's option to purchase the smaller company's output. Normally, the option provision would be illusory, because the oil producer's promise to purchase, at its option, 100% of the smaller company's output is not a promise to do anything and therefore is not consideration. However, there is other consideration here—the oil producer's purchase of the smaller company's refining facilities. The contract must be considered as a whole, and here the oil producer promised to purchase the smaller company's refining facilities. Moreover, the specification of the quantity of 100% of output is sufficiently certain and can be enforced because the quantity of output can be estimated from prior output. The specification of the price of "current market prices" is also sufficient. Thus, the written agreement is enforceable in its entirety, and (A), (B), and (C) are incorrect.

A distributor of electric toy trains and a hobby shop owner entered into a written contract providing that the distributor will tender to the shop owner four dozen of a popular electric train set at a price of $100 apiece, to be delivered no later than October 31, to take advantage of the holiday shopping season. The shop owner chose to order from this distributor because its price for the train set was lower than that of other distributors. Shortly after the shop owner placed his order, the distributor raised its prices due to a sudden surge in popularity of that train set. Because the distributor did not have enough train sets to accommodate everyone due to the surge of orders, it decided to deliver train sets only to those buyers who had ordered them at the increased price. The distributor notified the shop owner that it would not deliver the train sets it ordered. The shop owner filed an action to force the distributor to deliver the train sets at the agreed-upon price. Will the court compel the distributor to deliver the train sets to the shop owner? A) No, because a contract for the sale of goods is not subject to specific performance. B) No, because the shop owner can buy them from another distributor. C) Yes, because the shop owner will not be able to buy them from another source at the contract price. D) Yes, because time is of the essence.

No, because the shop owner can buy them from another distributor. Because the shop owner can cover (i.e., buy the train sets from another source), a court will not grant specific performance. If the seller fails to deliver goods under a valid contract, the buyer has a number of remedies available, including the right to cover and the right to obtain specific performance if appropriate. A buyer may obtain specific performance of a contract for the sale of goods if the goods are unique or in short supply, but that does not appear to be the case here because the other distributors carried that train set. Thus, the shop owner can buy the train sets from another distributor and get the difference between the cost of the substitute goods and the contract price. Thus, (B) is correct. (A) is incorrect because, as discussed above, under certain circumstances a seller of goods may be subject to specific performance. (C) is incorrect because, as discussed above, the shop owner can buy the train sets from another distributor and then sue for damages for the difference in cost. Thus, specific performance is not the appropriate remedy. (D) is irrelevant to whether specific performance is granted and is unsupported by the facts.

A newsstand owner leased space in an office building. An investor purchased the office building and told the newsstand owner that he wanted to negotiate a new lease. During the negotiations, the newsstand owner and the investor orally agreed that the newsstand owner would have the exclusive right to sell newspapers and magazines in the office building. The investor prepared a written lease outlining the agreement, but forgot to include the agreement that the newsstand owner would have exclusive rights in the office building. The newsstand owner was given a copy of the lease to read, but he merely glanced over the lease because he assumed that it reflected his agreement with the investor. The newsstand owner then signed the lease, which included a merger and integration clause. One month later, the investor leased space in the building to a drugstore. The drugstore's lease did not prevent it from selling newspapers or magazines. As a result of the competition, the newsstand owner lost substantial profits in his business. The newsstand owner brings suit to reform the contract to reflect his exclusive right to sell newspapers and magazines in the office building.The most likely result of this suit is that: A) The newsstand owner will prevail. B) The newsstand owner will prevail only if he can prove a mutual mistake. C) The newsstand owner will not prevail because of the application of the parol evidence rule. D) The newsstand owner will not prevail because the mistake on his part was unilateral.

The newsstand owner is likely to prevail only if he can prove a mutual mistake. A contract can be reformed to reflect the original intent of the parties where there has been a mutual mistake in the integration. The plaintiff's negligence is not a bar to reformation. Here, both parties were unaware that the written contract did not reflect their agreement. (A) is not as good an answer as (B) because it fails to mention the mutual mistake element. (C) is wrong because the parol evidence rule does not apply in an action for reformation; if it did, contracts could rarely be reformed. (D) is wrong because the mistake here was not unilateral. Both parties believed that the written lease contained the exclusivity provision.

A writer contracted with a literary agent to obtain a publisher for his book. The parties' written agreement provided that the agent would receive 20% of what the writer earned from each copy of the book sold. The agreement also provided that the publisher procured by the agent would pay the writer an advance against royalties of at least $10,000. The agent found a willing publisher who entered into a written contract with the writer providing for the standard royalty amount minus the $10,000 that the publisher promised the writer as an advance. The writer insisted that a clause be added to the contract providing for liquidated damages of $50,000 if the publisher breached the contract. Although the writer was unsure what his actual damages would be, he thought he could make $50,000 in royalties. The writer and the publisher signed the contract, and the publisher paid the writer $10,000 per their agreement. A few weeks later, the publisher notified the writer that, due to unexpected financial constraints, it was no longer possible for it to publish the book. The publisher requested that the writer return the $10,000 but the writer refused, so the publisher filed suit. The agent intervened in the suit, demanding the $10,000 from the writer as her 20% share of the expected royalties. Regarding the $10,000, what are the parties contractually entitled to? A) The writer is entitled to keep the $10,000 because the liquidated damages clause is reasonable. B) The publisher is entitled to get the $10,000 returned because its contractual duties were discharged by impossibility. C) The agent is entitled to get the $10,000 because she performed her part of the contract by finding a publisher. D) The writer is entitled to keep $8,000 and the agent is entitled to get $2,000, because the liquidated damages clause was reasonable and the agent is entitled to 20% of whatever the writer gets from the deal.

The writer is entitled to keep the $10,000 as liquidated damages. A liquidated damages clause will be enforceable if: (i) damages are difficult to ascertain at the time of the making of the contract, and (ii) the damages are a reasonable forecast of compensatory damages. The test for reasonableness is a comparison between the amount of damages prospectively probable at the time of contract formation and the liquidated damages figure. Here, the damages were difficult to ascertain at the time the contract was entered into, but $50,000 appears to be a reasonable forecast of damages by the writer given the amount of the advance. Thus, the writer is contractually entitled to keep the $10,000 as part of his liquidated damages. (B) is incorrect because "impossibility" will discharge a contract only when performance cannot be accomplished by anyone. Here, the publisher may have been required to find other resources to get the book published or subcontract the job to another publisher. Thus, it likely was not impossible to publish the book. (C) is incorrect because the contractual condition for the agent's payment was not met: She was to receive 20% of the amount of money the writer earned from the sales of his book if the book were published, and the book was not published or sold. (D) is similarly incorrect because the agent was entitled under her contract to receive proceeds from the sales of the book if it were published; the book was not published and the $10,000 was liquidated damages and not proceeds from sales.

On July 1, a farmer in the business of raising and selling tomatoes sent the following signed letter to a produce wholesaler, which was received July 2: "This is an offer for 1,000 bushels of tomatoes available for delivery at $10 a bushel. This offer will remain open until July 15." On July 5, a severe thunderstorm destroyed the tomato crop in a neighboring county, causing the wholesale price of tomatoes to rise to $20 a bushel. On July 6, the farmer e-mailed the following statement to the wholesaler: "We had to raise our prices. Update our offer of July 1 to include the new price of $20 a bushel." The e-mail was received by the wholesaler on the same date. On July 12, the wholesaler wrote to the farmer: "I accept your offer of July 1. I would appreciate you delivering 1,000 bushels on August 15." The farmer received the wholesaler's letter on July 14. No further communications took place between the parties. On August 15, what is the status of the contractual relationship between the parties? Question 9 options: A) There is no contract, because the farmer's offer was revoked prior to the time it was accepted. B) There is no contract, because the purported acceptance stated terms at variance with the offer. C) There is a contract for delivery of 1,000 bushels of tomatoes on August 15 at the price of $10 a bushel, because the farmer's offer was a firm offer and the wholesaler's letter was a proper acceptance of that offer. D) There is a contract for delivery of 1,000 bushels of tomatoes on August 15 at the price of $20 a bushel, because the farmer has properly updated its offer and the wholesaler's letter was a proper acceptance of the updated offer.

There is a contract for delivery of 1,000 bushels of tomatoes on August 15 at the price of $10 a bushel, because the farmer's offer was a firm offer and the wholesaler's letter was a proper acceptance of that offer Answer C is correct. This is a contract for the sale of goods and is governed by the UCC. Under Article 2, if a merchant offers to buy or sell goods in a signed writing and the writing gives assurances that it will be held open, the offer is not revocable during the time stated (or a reasonable time if no time is stated) except in no event may the period exceed three months. Here, the farmer is a merchant and in a signed writing gave assurances that the offer would be held open until July 15. Thus, the farmer could not revoke the offer before that time, and the wholesaler's acceptance on July 12 was valid. Under the UCC battle of the forms provision, an acceptance is valid even if it contains additional or different terms. Between merchants, additional or different terms will be included in the contract unless they materially alter the original terms of the offer; the offer expressly limits acceptance to the terms of the offer; or the offeror has already objected to the particular terms, or objects within a reasonable time after notice of them is received. Here, the wholesaler merely stated that he would appreciate two separate deliveries, if possible, on August 1 and August 15. This is merely a statement of preference. The original offer clearly stated that August 15 was the earliest possible harvest date for the tomatoes so they would not be available on August 1. Thus the wholesaler's statement of preference would not become part of the contract.

A landowner sent a signed letter to a potential buyer, offering his property for $100,000. The buyer received the letter and sent a signed letter to the landowner, stating: "I accept your offer to sell me your property for $100,000." The buyer, however, then sent another letter on the same day by overnight mail to the landowner, stating: "I will buy your property, but I can only pay $90,000." The landowner received the second letter and then received the first letter. The next day, the buyer then sent a third letter which stated: "I will pay the $100,000 that I originally offered for the property." The landowner received this letter and wrote to the buyer that she would not sell the property to him. The buyer then tendered $100,000, but the landowner refused to tender a deed. If the buyer sues the landowner for specific performance, will he succeed? Question 2 options: A) Yes, because a contract was formed at the moment the buyer sent the first letter. B) Yes, because the buyer's third letter resulted in an enforceable contract. C) No, because the second letter sent by the buyer was a rejection. D) No, because the counteroffer reached the landowner before the acceptance.

Yes, because a contract was formed at the moment the buyer sent the first letter. Answer A is correct. Under the mailbox rule, a contract is formed when the acceptance is properly dispatched, i.e., when an offeree deposits an unqualified acceptance in a mailbox. The buyer's second letter is a counteroffer, which at common law is a rejection. A rejection is effective upon receipt. If the offeree sends the acceptance first and then a rejection, the mailbox rule still applies. However, if the offeror received the rejection first and changed his position in reliance on it, the offeree will be estopped from enforcing the contract. Here, there is nothing in the facts to suggest that the landowner changed his position in reliance on the rejection; e.g., he has not entered into a contract to sell the property to someone else. All of the letters were sent within a two-day period, so there was very little time for the landowner to change his position. Absent a showing of detrimental reliance on the rejection, the contract formed when the buyer posted his first acceptance is enforceable.


Kaugnay na mga set ng pag-aaral

Exam 1: Chem 1.5-2.7, Chem 3.1-3.5, Chem 1.1-1.4

View Set

1 JRE, JDK, JVM || The Life Cycle of a Java Program

View Set

Statistics Chapter 5: z-scores and standardized distribution

View Set

The Picture of Dorian Gray - El Retrato de Dorian Gray (Oscar Wilde)

View Set

Religion 101 - 1:Religious Repsonses

View Set

Major #4 - EMT (Multiple Choice Questions)

View Set

PrepU Chapter 50: Assessment and Management of Patients with Female Physiologic Processes

View Set

Business Exam Number 1 Chapters 1, 4, and 7

View Set

Diagnostic Imaging: X-ray vs CT vs MRI vs Ultrasound

View Set

Developmentally Appropriate Care of the Adult Patient

View Set